Sei sulla pagina 1di 67

1

Torts I
Professor Nelson
Introduction to the Course: This course will cover three main types of torts: intentional,
negligence, and strict liability. Each tort has elements, but the difficulty is applying the
facts to the rules. State law will be the focus, because torts do not often become federal
issues. Finally, public policy will likely play a very large factor in the court decisions,
even if it is not even alluded to in the opinion.

1. Know the Black Letter Law for intentional torts, negligence, and strict liability.
Intentional Torts
Battery
Assault
False Imprisonment
Intentional Infliction of Emotional Distress
Conversion
Trespass to Chattels
Trespass to Land

Defenses to Intentional Torts


Consent
Insanity
Self-Defense
Defense of Property
Recapture of Chattels
Necessity

Negligence

Defenses to Negligence (and Strict Liability)


Contributory Negligence
Assumption of the Risk
Comparative Negligence

Traditional Strict Liability Actions


Animals
Ultrahazardous or Abnormally Dangerous Activities
Private Nuisance
Public Nuisance

Recoverable Elements of Compensatory and Punitive Damages

2 Objectives of Tort Law


Corrective Justice – to correct wrongs. The belief that payment of money damages by
the D to P will make the injured P whole again. Fairness for its own sake.
vs.
Law & Economics Movement – focus on incentives to reduce the costs of accidents

1
2

I. Intentional Tort

A. Battery (intentional invasions of other’s right of personal


security)

1. The elements:

a. Voluntary act

b. Intention to cause harm or offensive contact or imminent


apprehension thereof

c. Actual harmful or offensive contact directly or indirectly occurs

2. Extras:

a. Battery and intentional Torts, in general, are intended to narrow down fact patterns
that have a higher level of culpability than mere carelessness. Negligence will facilitate
needs in that area.

b. Intent is the most ambiguous piece. A person may have the best of motives (fix a
broken arm, clean their apartment by throwing out water from the fourth floor) and still
commit a battery.
Intent also transfers. If I intended to hit Bill but missed and hit Bob with a shotgun, then
I have assaulted and battered Bob.

The question to ask is did the offender know with substantial certainty the he would
cause the harm or offense? (Ex. of this is the Workman’s Comp work around where the
attorney proves that the employer knew with substantial certainty that removing a piece
of safety equipment would result in an injury.) Intent can also be described in terms of
simple (meant to intend the contact regardless of what the contact should have been
assumed to produce), actual (hardly ever will admit to it though), or implied (substantial
certainty).

*Prof. comments – “Intent is driven by policy concerns. How we define intent is


determined by circumstances.”

c. Harm can basically be anything that physically impairs the


body or causes physical pain or illness (Restatement) and, even if it is not harmful, it is
still tortuous if it is offensive or would be offensive to a “reasonable sense of personal
dignity” (Restatement). The question is whether a reasonable person in that situation
would be offended, and this takes into account social norms (ex. Customs) and
relationships (buddies that slap each other all the time, a prior course of contact).

d. Contact (volitional act) does not necessarily mean person to


person or at that exact moment (pokes with a pole, sets up a trip wire). The contact

2
3

requirement also includes objects intimately associated with the victim’s body, i.e.
pulling coat lapels, yanking a book out of someone’s hands, hitting them while they’re in
their car, etc. Consequences and contact are different, but the offender is liable for the
consequences of the battery.

3. The cases:

a. Garrett v. Dailey 1955 (CB page 7, Wash. S.C.)

1. Five year old boy moves chair and woman attempts to sit and damages her hip.

2. Regardless of the subjective motivation of the offender, he or she is still liable if


the actor was substantially certain that the action would cause harm or offense.

3. Extra: The court was concerned with Brian’s age regarding whether a five year old
could be substantially certain that moving the chair in the first place would result in harm,
not the reasonable man standard. So are we concerned with the mental state of the
offender in determining intent with regards to substantial certainty? What if the
guy was drunk, tired, elderly? Would that affect substantial certainty? Also, parents can
be held liable for the torts of their kids: limited liability (only liable if they themselves
were negligent in failing to supervise the children.

*Adults of diminished capacity have been held liable based on intent as long as they are
capable of formulating in their mind intent.

b. Vosburg v. Putney
ISSUE:
• Is the boy liable for causing harm, even though he didn’t do it intentionally?
HOLDING:
• The court found that the boy was liable, despite his lack of malicious intent.
RULE:
• Both intentionally and unintentionally harmful acts can be penalized under the
law. If you intend to do an unlawful act, it’s a tort.
ANALYSIS:
• Defendant contends that he shouldn’t be held liable for kicking the boy because it
wasn’t a criminal act. He didn’t intentionally kick the boy to hurt him, therefore
he can’t be liable. But the jury believes the boy is liable because, even though he
couldn’t foresee the harm he caused, he still caused serious harm by kicking the
child in the leg.
Battery: Intentional harmful or offensive contacts
Assault: An apprehended offensive, harmful contact

c. Talmage v. Smith
1. A man threw a stick at a couple of boys who were on his property. He didn’t hit the
boy he was aiming at but hit another, causing him to lose his eyesight permanently. Is

3
4

this battery when he did not intend to hit the boy he actually hit? There is also a privilege
to run the boys off the property v. reasonable force applied element to this case.

2. Intent may be transferred in the event that the offender fails to harm his target
yet harms another – the act is just as culpable when the aim is bad as when it is
good.

*Intent to commit one of five intentional torts (battery, assault, false imprisonment,
trespass to land, or trespass to chattels) is sufficient to make out intent for any of the
others.

3. Extra: Could this have been an assault on the boy he missed and battery on the other
boy? What if Smith had intended only to scare them off his property and not hit either
boy? In this case, throwing a stick to scare the boys is probably reasonable since they are
on his property.

In order that a contact be offensive, it must have been of a nature that would offend
a reasonable person.

A battery can exist even if the offender did not directly touch the offended, as long
as the offender clearly invaded the person of the offended.
3. Extra: This was also likely policy, given the year. The scope of employment issue
may have gone either way. This might have also been intentional inflection of emotional
damages.
*The court has held that there are some items so intimate that they are extensions of one’s
person – clothing, a cane, or anything held in the hand.
Alcorn v. Mitchell (p. 63) (Offensive Battery))
ISSUE:
• Is spitting on someone a battery.
HOLDING/RULE:
• Sure is.
ANALYSIS:
• It made it worse that the offense happened in court room.
• Also, defendant was rich, so judge said he’d have no problem paying the $1,000.
• Court said act was purely malignant and deserved to be stiffly punished.
• Perhaps punishment was stiff to discourage people from fighting something out in
a duel.

For Battery, there has to be intent and actually directly or indirectly


make CONTACT – can be anything – a poke

4
5

B. Assault (protects the victims rights to be free of meaningful threats or


unwanted touching)

1. The elements: (Restatement 2nd Section 21)

a. voluntary act

b. Intention to cause harm or offensive contact or imminent


apprehension thereof

c. the other or a third person is thereby put in such imminent


apprehension

2. Extras:
Assault is basically battery without or even before the contact,
although there is not always an assault when there is a battery (for example,
kissing someone while they’re sleeping, no imminent apprehension).

To constitute assault, it will be necessary to determine if a


reasonable person would consider the contact harmful or offensive. If it would not have
been either, then the plaintiff will have no claim for the imminent apprehension
(perception or anticipation of a blow).

Imminent Apprehension is tricky – it needs to be able to happen, not necessarily at that


moment, but quickly. The Restatement advises: it is not necessary that one shall be
within striking distance of the other, or that a weapon pointed at the other shall be in a
condition for instant discharge. It is enough that one is so close to striking distance that
he can reach the other almost at once, or that he can make the weapon ready for discharge
in a very short amount of time. The offended only needs to perceive the reality of the
threat as a reasonable person would (pulls a very real looking plastic gun example).

Words alone do not usually constitute assault unless they are together with acts or
circumstances that would put the other in reasonable apprehension of an imminent
harmful or offensive contact with his person (Restatement). Circumstances may mean
some serious body language or threatening behavior.

Conditional Threats vary on whether they constitute an assault but, generally, if the
condition is something in the past (if you hadn’t, I would harm you) then it’s not assault
because there’s no imminent apprehension, however, if the condition is present, (your
money or your life) then it is probably assault. What about future?

5
6

C. False Imprisonment

1. The Elements:

a. volitional act

b. intent to confine within boundaries

c. actual confinement within the boundaries

d. the person confined must be conscious of confinement or


actually harmed

2. Extras: confinement is ambiguous and the big one here because often
the person thinks that the confinement in implied while the person who may be implying
it suggests that the other person was free to go at anytime. The shopkeeper’s privilege
comes up a good deal in terms of shop lifting, which often turn the false imprisonment
(intentional tort) case into a case of negligence. If there was no statute, then the common
law dictated to that the shopkeeper acted at their own peril and could be sued for false
imprisonment if they guessed wrong. If the person is not conscious of the confinement it
didn’t happen unless that person was actually harmed. Implied threat and not stated can
be a case of false imprisonment.

*Shopkeeper’s privilege has three components:


1. Reasonable belief person has stolen or is stealing something
2. Detention for a reasonable time
3. Detention in a reasonable manner

3. The Cases.

In order for a threat to mean detention, the threat must provide a just fear of injury
to person, reputation, or property. And the shopkeeper must be reasonable in its
detention (in a reasonable manner for a reasonable time).

*The court said in this case that physical restraint is not the only way to establish
willful detention.

Bird v. Jones (p. 65) [false imprisonment]


115 Eng. Rep. 688 (K.B. 1845)
FACTS:
Pl. could have gone any other way but the way he wanted to go. He sues for false
imprisonment.

ISSUE:
• Is prohibiting a man from going in one direction, when he can go in other
directions freely, false imprisonment?

6
7

HOLDING/RULE:
• The court held that it wasn’t false imprisonment since the man had other options
as to places to go. Also, the def. and his men didn’t threaten him or keep him
physically from leaving.
ANALYSIS:
• The court didn’t want to make false imprisonment so broad as to apply to any
barring of a public right of way, specifically when there are other avenues of
travel open.
• Also, they made it clear that the imprisoner must physically keep the person from
leaving or threaten the person with harm if they try to leave.
• The dissent found that preventing a man from going wherever he wants to is false
imprisonment.
• Must have confinement to have false imprisonment.

Coblyn v. Kennedy’s, Inc. (p. 68)


268 N.E.2d 860 (Mass. 1971)
FACTS:
• Coblyn was a 70-year-old man who was shopping at Kennedy’s. In order to try on
some clothes, he took off his scarf and put it in his pocket. He purchased a sport
coat and left it for alterations. On his way out, he took his scarf back out of his
pocket and put it on. At this point a security guard came up, grabbed him by the
arm in front of several people, and told him he had “better go back and see the
manager.” The trip up the stairs so winded the man he had to go to the hospital
for heart problems.
PROCEDURE:
• Coblyn brought suit, jury awarded him $12,500 for false imprisonment. Appeals
court overruled the def.’s exceptions.
ISSUE:
• Can store security be charged with false imprisonment for detaining suspected
thief, if person turns out not to be a thief?
HOLDING/RULE:
• Court holds that man was falsely imprisoned since guard grabbed him by the arm
in front of several people and told him to go along with him.
• Court holds that shopkeepers have to have reasonable grounds to detain someone.
ANALYSIS:
• There was no way the old man could have escaped. Also, to just walk out of the
store would make the others in the store think he was a thief.
• Court also found there were no reasonable grounds for thinking the man was a
thief.
• Court didn’t want to give store security more power than police by letting them
act on reasonable suspicion. Police have to act on probable cause.
• Shopkeeper had to show reasonable grounds and honest suspicion that person is
shoplifting before he can detain someone.
• Negligence standard. (Reasonable business)
• Incentive for shopkeepers, security staff to not detain innocent people.

7
8

• But, privilege still there, since shoplifting is bad.

Parents, guardians are able to do anything reasonably necessary to discipline children.


Also, can prevent children from damaging property. (p. 71)

D. Intentional Infliction of Emotional Distress

1. The Elements:

a. volitional act

b. extreme and outrageous conduct

c. intentional or reckless

d. a causal connection between conduct and the severe


emotional distress

e. actually cause severe emotional distress.

2. Extras: Also called the tort of “outrage,” the courts are very strict about
granting damages of IIED by itself. Before this tort was made independent, it was often a
part of damages in other cases.
Extreme and outrageous conduct means that “it goes beyond all bounds of human
decency.”
Intentional or reckless conduct means that it has to be intended to cause the stress, a
person would be substantially certain that it would cause it, or a person is reckless and
disregards that the action will likely result in emotional distress. There has to be a way to
prove that the distress is severe and was caused by the conduct. Court differs on how to
do this, some require physical harm to occur (vomiting), some don’t.
This tort does not require any physical contact, but it is not intended for a simple
insult. Exception: The special carrier/utility rule: common carriers and public utilities
have been held liable for gross insults that would not otherwise be actionable under the
common law requirement of extreme and outrageous conduct. Race or gender
harassment may be merely an insult and still constitute IIED and, in the event that a
family member is around when it happens, the offender may also be liable to that person.
Courts try to avoid hearing this in connection with marriage. Transferred intent does
not apply to this tort.

3. The Cases.
Wilkinson v. Downton (p. 72)
[1897] 2 Q.B. 57
FACTS:
• Downton went to Wilkinson’s residence and told her, falsely, that her husband
had been seriously injured was lying at The Elms at Leytonstone (hospital) with
both legs broken, and that she should immediately fetch a cab and go to him. Pl.

8
9

suffered severe emotional distress and became violently ill with worry. The
effects lasted a considerable amount of time past the actual day of the offense.
PROCEDURE:
• Pl. sued for fraud and deceit, cause of action.
• Trial jury ruled for pl. and awarded money it cost for the pl.’s friends to fetch a
cab to get her husband. They also granted 100 lira for injuries caused by nervous
shock.
• Def. claimed that pl. shouldn’t be able to recover for shock since at the time it
wasn’t actionable. Appeals court affirmed trial court’s ruling for pl.
ISSUE:
• Is mental distress actionable?
HOLDING/RULE:
• The court holds that the def,’s actions were full of malice, and that he intended to
cause distress, though perhaps not at the resulting level. This, of course, is no
excuse. Pl. can recover for emotional distress though court notes that it is setting a
precedent by doing so.
ANALYSIS:
• Court wanted to make clear that intentionally imposing emotional distress is a
tort.
• Court awarded damages for mental distress, but don’t call it that – allow damages
as “parasitic” on fraud and deceit charge.

Restatement gives some rigorous requirements for emotional distress. Don’t want too
broad cause of action.

Potts v. Hayes Printout Case

Howell v. New York Post Co.

9
10

10
11

E. Trespass to Land

1. The elements:

a. Volitional

b. Intent to perform the act that results in trespassing

c. Enter the land of another person.

2. Extras: “One who intentionally enters land in the possession of another


has committed the tort of trespass to land. Even one who mistakenly steps on another’s
property, believing it to be his own, commits the tort.” But…not when they didn’t have a
choice in the matter such as losing control of the vehicle and ending up in the front yard.
A possessor of real property has a right to exclusive possession. No damage is required.
Transferred Intent doctrine does apply to trespass to land.

3. The Cases:
Dougherty v. Stepp (p.9)
18 N.C. 371 (1835)
FACTS:
• Def. went onto Pl. land to do some survey work. He didn’t mark any trees or hurt
anything, but he was on the Pl. land without permission. Def. was also trying to
claim the land as his.
PROCEDURE:
• Pl. brings suit in Circuit Court saying that the Def. should be penalized for
venturing onto his land without permission. Judge rules for def.
• Pl. appeals, and appeals judge reverses decision and rules for Pl.
ISSUE:
• Is it unlawful to be on someone else’s property, even if you do no harm to it?
HOLDING/RULE:
• It is unlawful to be on someone else’s property without permission whether or not
you harm anything.
ANALYSIS:
• The appeals judge wanted the act of invasion on another person’s land to be the
main issue, whereas the trial judge got bogged down in whether or not harm was
caused. Harm is beside the point; it’s the invasion of private property that matters.
Protecting private property titles. Strict liability?

2. Leaving a piece of property, depending on the statute in that jurisdiction, on


someone’s land may constitute a trespass if the consent has expired or been
removed.
3. Professor comment: Sometimes in these intentional torts cases, we’re making it
look like strict liability (trespass) and negligence (shopkeeper) because we’re
stretching.

11
12

F. Trespass to Chattels

1. The Elements:

a. Volitional Act

b. Intention to perform the act that constitutes the trespass

c. dispossessing someone of personal property or intermeddling


with or using the property.

2. Extra: Trespass to Chattels deals with intentional interferences with the personal
property of others. It’s the lighter version of conversion.
• The plaintiff may only recover the amount of harm done to the chattel in a
dispossession case (including rental fees or whatnot if the plaintiff suffered in its
absence).
• In an intermeddling case: No harm, no money. Example was taking a book for an
hour and spilling coffee on it, defendant should pay for the coffee spill. If she
takes the book for an hour, it may be conversion and the ▲ should pay the entire
value. Transferred intent doctrine applies.

3. Cases:
a. Compuserve Inc. v. Cyber Promotions
1. Cyber promotions made a business of spamming compuserve’s email
account holders and was persistent despite a request to discontinue and technological
attempts to thwart the spam. Compuserve sued for trespass to chattels.
2. Electronic signals may constitute physical contact in regards to trespass to
chattels; under the Second Restatement, recovery may be possible for intermeddling
if the proper criteria is met.
*The court also assigns liability under the Second Restatement because that chattel was
“impaired as to its condition, quality or values.”

Intel Corp. v. Hamidi (p.13)


71 P.3d 296 (Cal. 2003)
FACTS:
• Hamidi is an Intel employee who sends out emails criticizing the company to its
employees on six separate occassions. He removes anyone from the list who
requests it. Intel gets pissed and files suit. Emails didn’t hurt anything.
PROCEDURE:
• Intel files suit against Hamidi claiming he committed the tort of trespass to
chattels by using company emails. Trial court allows pl. to have summary
judgment, silencing Hamidi. Supreme Court disagrees.
ISSUE:
• Does sending emails within a company that the company doesn’t like constitute a
trespass?
HOLDING:

12
13

•Appeals court holds that electronic communication doesn’t constitute an actual


trespass to personal property. He did nothing to harm the company, the
computers, or the email system.
ANALYSIS:
• There is no trespass against Intel since Def. didn’t interfere with possessor’s
possession of the personal property.
• Dissent says that the real issue is that Intel’s network is personal property, and
just because he didn’t crash the system didn’t mean that he wasn’t doing
something harmful to the system. Epstein wants it treated as real property. But the
pl. has to prove actual damage if it’s treated as real property. Same thing applies
to personal property, must show harm.
• Court doesn’t want to get into the idea of free speech, but would prefer the
legislature get into it.

G. Conversion
1. The Elements:

a. Volitional Act

b. Intention to perform the act that constitutes the trespass

c. exercising dominion or control of the property

d. serious interference with the other’s right to control

2. Extra: The real difference is degree of interference; conversion is market value or


harm caused while trespass to chattels is only the harm caused, if any. If the
defendant destroys or materially alters a chattel, there is a conversion.
An important factor is whether any harm to the chattel was caused by or during the
improper use. Buying or selling stolen goods is still a conversion because the buyer still
asserts ownership rights and deals with the chattel in a manner inconsistent with the
rights of the true owner.

*Book definition – conversion is the intentional exercise of dominion or control over a


chattel which so seriously interferes with the rights of another to control it that the actos
may justly be required to pay the full value of it.

a. Restatement Second of Torts 222A (2) says In determining the


seriousness of the interference and the justice of requiring the actor to pay the full value,
the following factors are important:

1. the extent and duration of the actor’s exercise of


dominion or control

2. the actor’s intent to assert a right in fact inconsistent


with the other’s right of control.

13
14

3. the actor’s good faith

4. the extent and duration of the resulting interference


with the other’s right of control.

5. the harm done to the chattel.

6. the inconvenience and expense caused to the other.

3. Cases: insert from cb 13-35


Examples: 1) If A intentionally destroys B’s chattel, A is liable for conversion.
2) If A intentionally caused minor damage to B’s chattel, A is not liable for conversion
but would be liable for trespass to chattel.
3) If A attempts to steal B’s chattel, but is caught within minutes, A is liable for
conversion because of the weight placed on A’s bad faith.
4) Purchasing stolen property even if purchaser is acting in good faith constitutes
conversion by both the seller and the innocent buyer.

H. The Umbrella Intentional Tort = Third Restatement of Torts §5; An actor


who intentionally causes physical harm is subject to liability for that harm. This would
cover down on some of the intentional wrongdoings that are a stretch under other torts.

14
15

II. Privileges/ Defenses

A. Consent

1. The Concept: consent means what it sounds; if the defendant can prove that the
plaintiff consented either verbally, by a manifestation of actions, or another way, then this
is an affirmative defense.
Exceptions including: if procured by fraud,
if the act goes beyond the scope of consent;
the person is consenting under a state of duress;
the defendant presents a mistake of fact or of law;
consent is not informed (think medical here);
lack of capacity to consent (infant, drunk, etc.), and
(differs by court but majority agrees) if the consent is to a criminal act.
Professor comment: If it’s an element of the case (no consent) then the burden is on the
╥, if it’s an affirmative defense, the burden is on the ▲. Is there some ambiguity here? T

2. The Professor’s take: there are two types of consent: valid consent and
informed consent. A cause of action surrounding a legit valid consent (a battery cause
of action) is rare, while informed consent often does not do well but is more common.
Informed consent often applies to the medical profession and can be either a battery or
negligence (more common) cause of action. Battery is easier to prove because there is no
need for an expert testimony.

1. General Rule: Consent is a defense to intentional tort liability. If the


victim gives permission, the conduct that would be tortious is
privileged. Consent is subjective.
 Violenti non fit injuria: to one who consents, no wrong is
done. Consent operates to eliminate the ∏’s prima facie case, the
offensiveness of the contact, rather than operating as a separate
affirmative defense.

2. Express Consent: Consent is a valid defense when it is objectively


manifested. An individual can express through words or gestures.

3. Implied Consent: Consent is implied when, under the circumstances,


the conduct of the individual reasonably conveys consent.

4. Restatement, Second, of Torts §892. Meaning of Consent


(1) ACTUAL CONSENT: Consent is willingness in fact
for conduct to occur. It may be manifested by action or inaction
and need not be communicated to the actor. (In the victim’s head
there must be consent.)

15
16

(2) APPARENT CONSENT: If words or conduct are


reasonably understood by another to be intended as consent, they
constitute apparent consent and are as effective as consent in fact.

O’Brien v. Cunard Steamship Co. (Apparent


Consent)
FACTS: Woman stood in line to be inoculated, held up arm.
Didn’t want to be inoculated.
RULE: If plaintiff’s behavior is such that it indicates consent,
regardless of her actual feelings, defendant cannot be held liable
for battery.
5. Implied-In-Fact Consent: A continuing, informal pattern of
interpersonal behavior may constitute implied-in-fact consent among those
involved, for the behavior to continue. Consent tacitly given by one
individual to another based on their shared history is implied-in-fact
consent. (patterns of practical jokes may imply consent to continue with
the jokes or tenant knowing that cars parked in front of apartment complex
get towed implied consent for hers to be towed if parked there)

6. Implied-In-Law Consent: Some categories of conduct allow the


courts to attach consent as a matter of law. These may include
participating in professional football. Consent is also implied as a matter
of law in emergency situations.

7. Invalidation of Consent:

(1) Incapacity: Children generally do not have the capacity


to give consent. An individual without sufficient mental capacity
due to insanity or retardation may not legally consent. Incapacity
can also be invalidated due to drug/alcohol intoxication.
(2) Action Beyond Scope of Consent: Consent is
invalidated if it goes beyond the consent manifested.

Hackbart v. Cincinnati Bengals, Inc.


FACTS: Pro football player hit after the play was over. Sued
for battery.
RULE: By participating in the game, the player has not
implied consent to tortious activity beyond the scope of the
game and outside the rules which results in injury.

Mohr v. Williams

16
17

Patient was anesthetized and the doctor decided that the left
ear was more important to operate on than the right ear.
Patient sued for battery and won.
ISSUE:
• Is it assault and battery if the pl. consented to the operation on the other ear? Can
consent be implied to the left ear?
HOLDING/RULE:
• Consent cannot be implied, and assault and battery can be claimed in this
instance. Def. touched pl. in such a way that was wrongful or unlawful, even
though he did so with no evil intent.
ANALYSIS:
• Court found that consent must be expressly given and cannot be implied in places
where it wasn’t expressly given.
• Kennedy v. Parrot reverses this decision (N.C. 1956) No way to know if
there’s an extra problem until surgery (encourages doctor to look around
and see if there are any problems he or she can fix) Also, consent forms

Kennedy v. Parrott
FACTS: During appendix operation, surgeon sees ovarian cysts, bursts them, causes
woman to contract phlebitis.
RULE: Where an internal operation is indicated, a surgeon may lawfully perform and it
is his duty to perform such operation as good surgery demands, even if it means an
extension of the operation further than was originally contemplated, and for doing so he
is not held liable for damages for an unlawful operation.
(3) Fraud: Consent is invalidated if it is induced by fraud
that misrepresents an essential aspect of the interaction.

De May v. Roberts
FACTS: Dr. brought stranger into patients home. He helped
with delivery even though he wasn’t a doctor or med student.
Patient was under impression he was in the medical field.
RULE: If plaintff gives consent as the result of deceit, the
consent does not preclude the plaintiff from an action after
finding out the truth of the circumstances.
(4) Duress: Consent procured under physical threat is
invalid. Economic pressure generally does not negate consent.

(5) Lack of Capacity: Common law “mature minor”


doctrine: courts measure a child’s age, ability, experience,
education, training, and degree of maturity in order to determine
consent. Some states statutorily proscribe an age of consent.

(6) Illegality: Majority of courts hold that a person cannot


consent to a criminal act, that the consent is always invalid. The

17
18

minority view and the Restatement § 60, 61 hold that a person can
consent to criminal acts for purposes of tort liability.

(6) Informed Consent: failure to tell the patient about the


risks and alternatives to the procedure and the nature of the
procedure. This is only a negligence cause of action. This involves
having a duty, a breach of the duty, causation, and damages. A
patient's knowing choice about a medical treatment or procedure,
made after a physician or other healthcare provider discloses
whatever information a reasonably prudent provider in the medical
community would give to a patient regarding the risks involved in
the proposed treatment or procedure.

 Consent does not transfer even to other doctors or


surgeons. If one consents for one person to perform the
procedure and then another performs it, valid consent is
not present and is a cause for battery
 Elements:

i. Duty

ii. Breach of Duty:


(a) Prudent Patient Standard: Whether a
reasonable patients would want to be informed of these risks. (OBJECTIVE) This is the
minority view.
(b) Professional Standard: Whether it is the
standard/custom of the profession to tell of these risks. This is the majority view.
iii. Causation
(a) Counterfactual Causation: ∏ would have
chosen no treatment or a different course of treatment had the alternatives and material
risks of each been made known to him (P’s words only).
(b) Actual Causation: A causal connection
between the patient’s injury and the doctor’s breach of a duty to disclose exists only
when the disclosure of material risks would have resulted in a decision against it (requires
expert testimony). If the patient (a reasonable person) would have chosen to go ahead
with the treatment had he been informed of the risks, the element of causation is missing.
iv. Damages- The risk must have materialized and ∏
must have been injured as a result of the treatment.
Scott v. Bradford
FACTS: ∏ says that doctor did not properly inform her of the risks of the operation.
RULE: In a medical malpractice action a patient suing
under the theory of informed consent must allege and prove
(1) physician failed to inform him adequately of
the risks

18
19

(2) if patient had been informed he would not


have had the surgery
(3) the adverse consequences that were not
made known did in fact occur and patient was
injured as a result of submitting to treatment.

The scope of a physician’s communications must be


measured by his patient’s need to know enough to enable
him to make an intelligent choice. Full disclosure of all
material risks incident to treatment must be made. A risk is
material if it would be likely to affect a patient’s decision.

Negligence has two causation factors: actual causation, a known or should have been
known risk of the procedure materialized and the patient was not informed of that risk
and counterfactual causation focuses on something that didn’t happen, what the patient
would have done if he had known the risk (some courts use the plaintiff’s own subjective
standard but the majority use the reasonable man standard). A form does not always fly
with consent, duress negates consent, but not economic duress, lack of capacity also
vitiates consent. Professor brought up hypothetical questions where kids fight (can they
consent, what about mature minor doctrine), consenting in illegal situations, sexual
intercourse (Rule of 7 [can’t consent], 7-14 [presumed can’t], 14-17 [presumed can
consent]).

2) if he had been informed of the risks, he would not have


consented to the treatment;

3) the adverse consequences that were not made known did


in fact occur and he was injured as a result of submitting to
the treatment.

i. Exceptions include: anxiety (the patient would have been far too upset and refused the
treatment and suffered as a result), emergency (applies to both valid consent and
informed consent, maybe the patient is drunk, unconscious, a kid, etc.), and common
knowledge (patient should know this anyhow.

B. Self Defense

19
20

1. The Concept: Self Defense is an affirmative defense to an intentional


tort; the focus seems to be the reasonableness and proportion of the force used.

2. The Professor’s notes: He made sure to emphasize the existence of a


duty to retreat when not in the home OR when the home is jointly owned by the person
responsible for the threatening conduct when considering the use of force intending or
likely to cause death. There is no duty to retreat when the force is reasonable and not
likely or intending to cause harm.

20
21

4. The Cases:

a. Courvoisier v. Raymond, 1896


1. Guy thinks he’s shooting someone who was possibly a threat to his life but ends up
shooting a cop, and he’s sued for battery.
2. The self defense, defense requires that the defendant acted honestly in using force
and that his fears were reasonable under the circumstances, and also as to the
reasonableness of the means made use of.
ISSUE:
• Can self defense be claimed if the threat is only perceived?
HOLDING/RULE:
• The court finds that whether or not the threat is real, if a person feels his life is
threatened and reacts with force, it is justified. It was a reasonable mistake for the
def. to think the cop was a rioter – cop was negligent to walk up on an armed
man. Negligence standard.
ANALYSIS:
• The court wanted to make clear that self defense is a defense as long as there is a
perceived life-threatening situation. Doesn’t matter if the perceived aggressor is
innocent.

Defense of Others: American courts have extended a privilege to actors who intervene
and use force to protect and defend others from threats and attack by third persons.
Professor’s hypo: A (old lady) is being helped by B (guy who’s carrying her purse) and
C intervenes and injures B thinking that he is attacking her. How does that work? There
is a mistake of fact here. C and B are both good Samaritans here. Is C liable? C uses
reasonable force here. If you’re wrong here, you gotta pay even if the mistake is
reasonable under the circumstances.

*Many states have enacted statutes against the common law right to resist an unlawful
arrest.

C. Defense of Property
1. Courts generally hold that law should place a higher value on life than
on property. (“You can protect your property but you can’t use deadly force unless
there’s threat to human life.”)

2. The Cases:
a. Katko v. Briney, 1971
1. Guy ties up a shotgun to a door in his abandoned house and a trespasser is shot.
2. One may use reasonable force in the protection of his property but one may not
use such means of force as will take human life or inflict great bodily injury.

Bird v. Holbrook (p. 40)


130 Eng. Rep. 911 (C.P. 1825)
FACTS:

21
22

• Pl. is a 19-year-old who was asked by a maid to go into another man’s garden in
the afternoon to retrieve a fowl that had escaped. He climbed over the back of the
garden wall and walked into a trap. He was shot in the leg above the knee,
presumably badly maimed. Boy had no idea there were spring guns set up, didn’t
see them.
• The garden’s owner had set up a spring-gun trap because someone had stolen
flowers and roots from the garden recently. Someone asked the def. if he should
put up a notice that the traps were there, but the def. was afraid that if he put up
notice, he wouldn’t catch the thief. (If his general thing was to deter, why not put
up sign? Because he wanted to injure and detain.)
ISSUE:
• Is def. liable for setting gun trap on his own land to hurt criminals?
HOLDING/RULE:
• The court holds that the setting of spring guns without giving notice, is an
inhuman act, and the def. must yield redress to the sufferer for causing harm.
ANALYSIS:
• It would seem that force is only justified when someone is trying to protect
themselves or another human, not when someone is trying to protect chattels (in
this case tulips) from being stolen.
• Judge makes the point that boy was only a trespasser and if the def. had been
physically present at the time of the trespass, he couldn’t have even lawfully taken
him into custody, much less wounded him.
• Policy argument from defense: if you impose liability on defendant then it’s
impossible for people to protect their property from a distance.
• CJ mentions religion. The law and religion intertwine in this case.
• Notice requirement.

D. Recapture of Chattels

The self help privilege allows chattel owners to use reasonable force to reclaim property
from a dispossessor as long as there is a “hot pursuit” factor or sense of immediacy i.e. it
was very recently discovered that the chattel has been dispossessed. Once the sense of
immediacy is lost, the privilege is gone. If the party is wrong, then they are liable under
common law. The shopkeeper’s privilege is mentioned as, often, an exception outlined in
statutes. Repossession is governed normally by the UCC.

Kirby v. Foster (p. 46)


22 A. 1111 (R.I. 1891)
FACTS:
• Pl. Kirby was an employee of a company where $50 was lost. The money was
taken out of his paycheck. Then the boss gave him some money to distribute to
the help. Pl. took $50 out of it and kept it for himself, giving the rest back to the
boss. Pl. had asked advice from lawyer about it and thought that it wasn’t wrong

22
23

to take the money. The boss got pissed that pl. pocketed the money and he and his
son tried to forcibly take it from him.
PROCEDURE:
• Jury filed for plaintiff in lawsuit, def. petitioned for new trial on exceptions to the
rulings and refusals to rule of presiding judge.
ISSUE:
• Can someone use force to retake a chattel that was peaceably given?
HOLDING/RULE:
• The court ruled that someone cannot take a chattel that was peaceably given by
force; breaching the peace is unacceptable in this instance.
ANALYSIS:
• Court finds that pl. didn’t trick them out of the money or steal it, and he thought it
was rightfully his, especially after seeking counsel about it.
• Basically, it’s not OK to hurt someone to take something back from them that was
peaceably given.
• Important to note that this idea carries over into landlord disputes. Landlords
cannot violently evict tenants, even if they haven’t paid. Must go through peaceful
court channels to settle dispute.
• The self-help remedy of recapture is allowed when one person wrongfully
obtained possession of the chattel either by force, fraud, or without claim of right.
• Any privilege of recapture must be exercised promptly – the so-called hot pursuit
requirement – or else it will be lost.
• A chattel can be recovered peacefully, no disturbance of public peace, do not
harm, touch person. (Repo rules) No judicial proceeding necessary to do so.

E. Necessity
2. The Cases:
a. Vincent v. Lake Erie Transportation
1. A boat was tied off to a wharf in the middle of a transaction when a terrible storm hit.
The boat was deliberately kept on the wharf and prevented from drifting, resulting in
damage to the wharf.
2. When the defendant has a choice to avoid damaging another’s property and not
and he chooses to damage the property, he may be liable for damages, even if the
choice he makes is out of necessity and reasonable under the circumstances.

*The courts have held that it is contrary both to the principal of law and morality to
privilege one person to harm another’s property so as to merely safeguard his own
possessions. POLICY!!!!
*The courts have generally not held liable the destruction of property to save a life or
lives.
Ploof v. Putnam (p. 49, necessity)
71 A. 188 (Vt. 1908)
FACTS:

23
24

• Pl., his wife, and two minor children were sailing on Lake Champlain in a sloop
when a storm came up. For safety, the pl. moored his boat to the def.’s dock on an
island the def. owned in the lake. The def.’s agent, a servant, then went and
unmoored the ship, causing the ship to be destroyed and its occupants thrown into
the water and onto the shore -- injuries.
ISSUE:
• Was pl. justified in trespassing in order to save his life, those of his family and his
property?
HOLDING/RULE:
• The court cites multiple cases that say yes, one can trespass on another’s land to
save property, and most especially, human life.
ANALYSIS:
• Court wants to uphold the fact that human life is more important than property
rights. If trespassing is necessary to save lives, so be it. Also, if someone
trespasses to save property, as long as they do no harm, that’s ok too.
• Exception to general rule that someone must have permission to go on another’s
property.
• Pl. could have used force to moor to dock in order to save lives, property. Def.
didn’t have to help.

General average contribution – whenever property has to be thrown overboard,


everyone on ship has to make up for the loss.

Vincent v. Lake Erie Transportation Co. (p. 51)


124 N.W. 221 (Minn. 1910)
FACTS:
• The Reynolds was moored at a wharf in the harbor of Duluth unloading its cargo
at about 10 p.m. Nov. 27, 1905. An extra severe storm rolled up shortly after all
the cargo was offloaded, so the crew moored the boat to the dock in order to keep
it from drifting away. The storm was too severe for it to go anywhere else, so the
captain decided to weather the storm tied to the dock, where it did $500 in
damage.
PROCEDURE:
• Pl. claims leaving the ship moored to his dock was negligent, and that the captain
should have moved the ship elsewhere. Def. claims that because mooring to the
dock was a necessity, he shouldn’t have to pay pl.
• Trial court finds for pl. Order affirmed.
ISSUE:
• Does trespasser, who trespasses out of necessity, have to pay up if they damage
the person’s land?
HOLDING/RULE:
• The court held that if someone trespasses out of necessity and damages a person’s
land, then they should have to pay up. Can’t preserve one’s own property to the
detriment of someone else’s.
ANALYSIS:

24
25

• Court doesn’t want to put burden on injured party.


• Because sailors renewed ropes, court doesn’t really agree with dissent. Majority
really doesn’t look at contract between ship and dock, letting ship moor to dock.
• Also, $500 damage to dock, versus $10,000 boat was saved. Def. profited, pl. lost
out.
• Dissent states that if the boat was lawfully moored to the dock at the beginning of
the storm, then it was inevitable that the boat would harm the dock, since the
storm prevented the ship from leaving.

Public necessity
A private or public individual isn’t liable for destroying public property for the greater
good of society – complete privilege gives people incentive to act for the greater good.
(keep fire from spreading, wartime – keep things away from enemy.)
Public necessity and just compensation
U.S. Constitution says government must pay for private land taken for public use. But
doesn’t always work that way. United States v. Caltex Inc. p.57. In police power or
regulatory cases, it seems the government doesn’t have to compensate. Kent State case.

H. Disciplining Children

1. Courts have been reluctant to strike down spanking children (parents or


teachers) as longs as the punishment is “reasonable in light of the seriousness of the
offense, the attitude and past behavior of the child, the nature and severity of the
punishment, the age and strength of the child, and the availability of less severe but
equally effective means of discipline.”

F. Insanity Defense
McGuire v. Almy (p.33)
8 N.E.2d 760 (Mass. 1937)
FACTS:
• A nurse, the pl., is hired to take care of an insane woman around the clock. She
even sleeps in the room next door to her for 14 months. The def. had been violent
at times, breaking things, but never doing any harm to the pl. April 19, 1932, the
def. has a violent attack and begins smashing things in her room. The def. then
told pl. and the maid that if either one of them came in her room that she would
kill them. Pl. called for the def.’s brother-in-law for backup because they wanted
to go into the room and take the broken bits away from her before she hurt
herself.
ISSUE:
• Can an insane person be held liable for an assault and battery?
HOLDING/RULE:
• The court holds that the insane def. can be held liable, but mainly because the def.
had the intent to harm the pl. She verbally told the pl. that she would attack if the
pl. came into the room, showing intent to harm.
ANALYSIS:

25
26

• Judge is basing the entire ruling on the deranged woman’s intent to harm. Her
being insane is irrelevant here because she clearly had a wrongful intent.
• Judge also wanted judgment to serve as a warning to the families of crazy people:
take care of your crazies, because if they injure someone, they are at fault.
• Judge wanted to limit insanity defense in civil cases.
• Insane person caused injury, therefore insano should pay for it. (fairness)
• Previous case law supports that an insane person is liable for his or her actions.
• In criminal law, you’re absolved of liability if you’re insane.

III. Negligence Time!

A. The elements:

1. Duty (determined by the court as a matter of law)

2. Breach of Duty (determined by jury, reasonable person standard)

3. Causation (must have both types)

a. Cause-in-fact (the plaintiff must prove a connection between the


defendant’s conduct and the damage)

b. Proximate Cause (a cause that is legally sufficient to result in


liability, usually policy based decision. A single negligent act may produce untold and
unforeseeable consequences…courts will usually deny a duty once the cause becomes too
remote)

4. Damages – there has to be some harm in order t recover

B. Extras:

1. Class: The elements of negligence, unlike intentional torts, bleed into


each other and are difficult to parse separately, and courts don’t necessarily follow these
concepts rigidly. The concept of negligence is very flexible.
Professor says that, although Learned Hand’s Risk/Utility Balance is not really used a
great deal, it does provide an economic approach to negligence.
He also thinks that another way to look at it (product liability kinds of negligence) is for
companies to consider damage that results from their conduct as a cost of doing business
and to budget it in, maybe pass it on to the consumer. Maybe that is another way of
handling it economically because the companies that cannot profit because there are too
many damages will go out of business.

26
27

Professor Comment: “Nobody really, in trial court, uses this learned hand formula. In a
products case, you’re going to be determining if there’s a reasonable alternative design.
That opens up the arguments as to what’s reasonable as far as the burden.”

Poverty is not usually considerable in determining whether someone is negligent (i.e.


they tried to take reasonable precautions but couldn’t afford it, etc.).

2. Hornbook, The Reasonable Person: Risks are sometimes necessary but


the reasonable person considers those risks in light of the utility of the conduct; RP also
considers the extent (gravity) of the risks, likelihood (probability) of the risks, and costs
(burdens) of the risk. This is a bit cold blooded but there is an economic analysis to the
formula. The jury won’t like be instructed on the Learned Hand formula, but they will
consider whether the RP would have taken the risk under the circumstances using the
aforementioned criteria.

a. Personal Circumstances (sometimes relevant):

i. Physical disability counts, and we expect those kinds of


people to need to take risks to function in society (blind guy walking the streets).

ii. Mental capacity does not count; if it did the cases would
be trying a person’s character and intelligence rather than their conduct. No exception
for the dumb or mentally ill.

iii. The RP child is measured by age, intelligence, and


experience unless the child was participating in some dangerous, adult activity.

b. External Circumstances (always relevant):

i. emergency doctrine – the fact that the offender had to


make a very quick decision may be considered under the circumstances.

ii. Customs are relevant, suggests that the conduct is


acceptable under the circumstances, although it is not bulletproof; custom may lag behind
for a number of reasons (seat belt example)

iii. statutory standards of care, the RP obeys the law

iv. expert or professional status may be taken into


consideration as a circumstance.

v. facilities or resources available are circumstances to be


taken into consideration when deciding what is reasonable (GP litigating anti-trust in
Kansas but not D.C.).

27
28

C. The General Standard of Care: Negligence (Risk/Utility) Balancing

1. Lubitz v. Wells

a. Father leaves golf club in back yard and boy, in his backswing,
hits another boy with it. The question is whether the father could be held negligent for
leaving the club in the backyard.

b. The law requires no duty to keep common things out of the


reach of children.

2. United States v. Carroll Towing Co.

a. A bargee left his tow tied down in the daylight hours. A tug
boat unfastened the barge and the re-fastened after moving some barges around. The
bargee’s barge came loose and resulted in damage to a tanker before sinking to the
ground.

b. Learned Hand Formula: The court holds that an owner’s


duty to provide against resulting injuries is a function of three variables:
1. The Probability of the event happening.
2. The Injury’s gravity.
3. The Burden of adequate precautions.
B < PI

If the burden of adequate precautions is less than the gravity of the resulting injury
multiplied by the probability of that occurring then there is negligence. If the
burden of adequate precautions is greater, then no negligence.
This is Risk/Utility Balancing.

3. Washington v. Louisiana Power and Light Co.

a. A guy who owned a large C.B. Radio with a very tall antenna
had previously come into contact with an uninsulated power line at this home and
suffered some injury. Years later, his antenna hit the wire again, and he died as a result.

b. Custom may have a significant impact on whether a


potential offender has been negligent. In this case, it was the power company’s
custom to handle the wires the way they were handling it. The court here, according
to the professor, does a “half-ass” utility balance under Learned Hand.

D. The Qualities of the Reasonable Person

1. Vaughn v. Menlove

28
29

a. A less than intelligent guy built a chimney in a hay-rick


thinking he would reduce the likelihood of a fire, even though is significantly increased
it. Everyone told him that he would be doing this, so the question becomes was he
negligent when he truly believed that he was doing a good thing in spite of what other
people knew to be true?

b. An offender’s actions must be judged in light of what a


reasonably prudent person would do under the circumstances. Objective standard
for mental capacity.

2. Delair v. McAdoo 1936

a. The plaintiff was passing the defendant in his vehicle and the
defendant’s tire blew out resulting in damage to the plaintiff’s vehicle. Professor raised a
lot of discussion over this case with questions like, “Am I supposed to check all of my car
parts every time I get into the car? What if something happens when I’m going down the
road? Is this a strict liability standard here?”

b. “The law requires drivers and owners of motor vehicles to


know the condition of those parts which are likely to become dangerous where the
flaws or faults would be disclosed by a reasonable inspection.”

3. Special Knowledge Standard: Courts disagree as to whether someone’s


expertise or special knowledge should require them to act more reasonably than the
prudent person under certain circumstances and whether they could be held liable for not
doing so.

4. Emergency Situations and the Reasonable Person

a. Cordas v. Peerless Transportation Co.

i. A criminal jumped into a cab driver’s vehicle and


pointed a gun at him, telling him to drive. Instead of driving, he jumped out of the
vehicle. The vehicle continued to move and struck a woman and her two children.

ii. The law does not hold one in an emergency to


exercise mature or reasonable judgment under circumstances where he has an
opportunity for deliberate action. Hornbooks says emergency doctrine is really about
taking the fact that the actor had to hurry up into consideration of the circumstances.

iii. Classroom Excerpt: Was this an area of the city that


was crowded? So was this just negligence or perhaps recklessness or maybe even an
intentional tort? Is this case the same as Vincent, in that case if you needed to use that
property then you would have to pay for it. Is the court treating this as a self defense case
then? The court talks about the rescue doctrine (you don’t have any duty to rescue
somebody). Didn’t he create the situation himself? Prof says that this is a self defense

29
30

thing here. Is there a privilege to use force against these people on the sidewalk? How
does this sudden emergency doctrine come into this? We take the circumstances into
account. Prof draws the hypo on the board with cliffs on each side of the road…if he
swerves either way he will endanger himself. 200 kids are killed as a result. The court in
this case is pointing towards no duty to rescue. The issue is whether juries should get
special emergency instructions and even the courts that approve them will not if the
actor’s prior negligence created the emergency.

5. Reasonable Person and Customary Practice

a. Trimarco v. Klein

i. Shower glass shattered and guy was injured. Custom


was to begin replacing the shower glass with shatterproof glass.

ii. Custom plays a part in determining standard of care


because it reflects the judgment and experience of the many; however, it is still a
part of the “reasonableness under the circumstances” standard for negligence, and
it is for the jury to decide what customs and actions are reasonable.

*The court rules that when a customary practice is couple with a showing that it was
ignored and that this departure was a proximate cause of the accident it may serve to
establish liability.

*The court also established in the TJ Hooper case (tug owners not customarily equipping
their tugboats with radios) (1932) that “there are precautions so imperative that even their
universal disregard will not excuse omission.”

b. Roberts v. State of Louisiana

i. A blind guy did not bring his cane and bumped into an
elderly man who fell and injured himself.

ii. As to his physical characteristics, the reasonable man


may be said to be identical with the actor in question…At the same time, the
conduct of the handicapped individual must be reasonable in light of his knowledge
of his infirmity, which is treated merely as one of the circumstances under which he
acts.

6. Mental Incapacity: The courts will generally not consider mental


incapacity a defense to failing to meet the reasonable person standard. A policy decision
here, incentive is to encourage caretakers to take more precaution and mentally ill people
present more of a danger than physically challenged people. However, “when the
defendant was a competent negligent wrongdoer and the mentally ill plaintiff failed to act
reasonably with regard to his own safety, there is good reason to allow the mentally

30
31

challenged person to recover.” So…some courts have this subjective standard with
regards to a mentally challenged plaintiff and contributory negligence.

7. Religious Beliefs and Negligence: The hot topic is whether a


defendant should have to pay for the extra damages that result when a plaintiff does not
get adequate treatment do to religious beliefs. Professor mentioned Eggshell Skull
Doctrine – if the injuries are ten times worse just because someone has a skull as soft as
an egg shell, oh well…the defendant is paying; no objective standard for what kind of
damage would be likely, only subjective what kind of damage occurred. Professor thinks
that this doctrine has something in common with religious beliefs in regards to
exacerbating damages.

8. Children and Negligence:

a. Stevens v. Veenstra

i. A 14-year-old kid was driving a car and lost control,


resulting in damage to the plaintiff’s vehicle. The issue was whether he should be judged
as a reasonable prudent man or “child.”

ii. When undertaking adult activities, minors may be


held to the same reasonable standard as adults.

*The court also reasons that, after considering the accidents that adults are involved in
while driving, it is illogical to think that the danger lessens when they activity is
undertaken by a minor with little or no experience.

iii. Professor raises questions like, “what is an adult


activity? Shouldn’t people see this kid coming in a driver’s ed vehicle?”

b. Restatement Third or Torts: Liability for Physical Harm

§ 10. Children
(a) When the actor is a child, the actor’s conduct is negligent if it does not
conform to that of a reasonably careful person of the same age, intelligence, and
experience; except that
(b) A child who is less than five years old of age is incapable of negligence;
and
(c) The special rule in Subsection (a) does not apply when the child is
engaging in a dangerous activity that is characteristically undertaken by adults.

c. There is also something called the Illinois Rule which a


minority of states follow: If the child is above 14 then the presumption is that the child is
able to reach the reasonable person standard. If between 7 and 14, the presumption is that
he or she is not, and under the age of 7, the child cannot be negligent.

9. The Standard Care for Professionals

31
32

§ 229A. Undertaking Profession or Trade


Unless he represents that he has greater or less skill or knowledge, one who
undertakes to render services in the practice of a profession or trade is required to
exercise the skill and knowledge normally possessed by members of that profession
or trade in good standing in similar circumstances.

a. Boyce v. Brown, 1938

i. One Doctor put a screw in a woman’s ankle and another


doctor took it out later, explaining that the damage she suffered resulted from the screw,
medical malpractice.

ii. Certain General Rules for Malpractice Laid Out:

1) A doctor should possess the skill of the average


physician in that profession and use that skill.
Rule 1. is really no good anymore, because average means that half of the group
is dangerous
2) He must have violated the recognized standard of
good medical practice in the community in which
he is practicing.
Rule 2. may still be good in some instances but is more likely to be looked at on a larger
scale than just that community now.
3) The medical standard for the community must be
proven.

4) No presumption of negligence is made simply


because the patient suffered a negative result

5) A medical expert is necessary unless a layman


can determine that the physician breached a
duty

6) There must be testimony from other physicians in


the community saying that they doctor in question
deviated from the standard.
Rule 3 – 6 are all about expert testimony, and they are still relevant. An expert must
testify as to the standards and reasonableness of those standards except in cases were a
layman may determine the what is reasonable (sponge left inside someone).

iii. Professor explained that it is all about the custom and the Boyce case had a hard time
because they had to get doctors from the “community.” Most courts have dropped the
“average” standard here..

E. Judicially Determined Standards of Care

32
33

1. Risk/Utility Balancing to set the Standard of Care (Doesn’t happen


much)

a. Helling v. Carey, Wash. 1974

i. The plaintiff’s eye doctors did not administer an eye


pressure test, and she developed glaucoma and practically went blind. Expert testimony
revealed that the standard of medical care was to NOT conduct the test. The court made
up it’s own rules.

ii. Adhering to professional standards does not insulate


the defendant from liability if a deviation from those standards would be
reasonable.

iii. Professor went to some effort to demonstrate how


wrong the court was here, even regarding the facts of the case. Excerpt: “This case
scared folks because now they would know what the standard was until the court told
them so. Not only was the standard not binding, but the court would decide for itself and
wouldn’t need experts. Some say that this decision caused the malpractice crises of the
mid 1970s.” No court really followed this articulation of the court’s own standard of
care. Professor also raised argument for the court’s actions in stepping in:
customs/standards might lag behind in certain areas, customs are different by locations
and need to be uniform. “Let’s distinguish between resources and technique, we can
expect technique to be the same even though resources will not be.” Professor states that
the court didn’t know what it was doing and just basically sympathized with the young
girl. ***Not a good case for precedent.

2. Courts Interpretation of Statutes and Negligence Per Se; If the statute


or regulation mentions civil liability, it is not negligence per se…the court has a choice as
to whether it should implement the legislature’s standard. We need to determine what
type of harm the statute or regulation is trying to prevent and what class of people it is
trying to protect to determine if it is relevant. We also need to determine if it is setting a
standard of care. There is also a recognition that there is an “excused” violation of
certain statutes. Arguments abound under licensing requirements, excuses, type of
harm/class of people, and causal link between the violation and the damages. Courts will
typically hold that a lack of a license doesn’t violate negligence per se.

From Professor: Two positions for reasonable care standard:


1. Reasonable prudent position
2. Customs

Restatement, Third, of Torts: Liability for Physical Harm


§ 14. Statutory Violations as Negligence Per Se
An actor is negligent if, without excuse, the actor violates a statute that is designed
to protect against the type of accident the actor’s conduct causes, and if the accident
victim is within the class of persons the statute is designed to protect.

33
34

Professor’s Hypothetical: key in the ignition statute that imposes a fine on a driver for
leaving a key in the car and thief hits a ╥ can the ╥ sue the guy that left the guy in his
car? Another one…a mental patient comes along – he’s escaped –steals the car and gets
into the accident defending himself. How would you use the “type of harm” or “class
of persons that the legislature was trying to protect?” He likes this analysis in this
situation.

a. Martin v. Herzog, N.Y. 1920

i. Martin’s buggy was struck by the defendant’s car. The


plaintiff claimed negligence on the part of the defendant for failing to remain on the
proper side of the road. The defendant claimed the plaintiff was negligent for failing to
have her headlights on, which was also a violation of a New York statute.

ii. If a statute sets a standard of care and the offender


breaches the duty to maintain that standard, there still must be a causal connection
between that breach and the defendant’s damages.

iii. Professor changed the facts into what if the buggy


driver didn’t have a license? There is an argument that it contributed to the damage
because the person wouldn’t have been driving or the license really had nothing to do
with the fact that the buggy hit the car.

The Martin case set out three views as to the role the offense of a statute may play in
negligence litigation.
1. Only evidence of negligence – to be considered by the court when deciding
whether there was negligence
2. A prima facie case of negligence – creates a presumption of negligence that can
be rebutted by proof that a reasonable person would have acted the way the actor
had (“on its face absent other evidence”)
3. Negligence per se – the violation of a statute is negligence (“in, of and by itself”)

b. Reque v. Milwaukee & Suburban Transport, Wis. 1959

i. Plaintiff sues the bus company here for negligence when


she was injured getting off the bus because it was more than twelve inches from the curb,
a violation of Wisconsin statute.

ii. § 288. When Standard Of Conduct Defined By


Legislation Or Regulation Will Not Be Adopted
The court will not adopt as the standard of conduct of a reasonable man the requirements
of a legislative enactment or an administrative regulation whose purpose is found to be
exclusively
(a) to protect the interests of the state or any subdivision of it as such, or
(b) to secure to individuals the enjoyment of rights or privileges to which they are
entitled only as members of the public, or
(c) to impose upon the actor the performance of a service which the state or any
subdivision of it undertakes to give the public, or

34
35

(d) to protect a class of persons other than the one whose interests are invaded, or
(e) to protect another interest than the one invaded, or
(f) to protect against other harm than that which has resulted, or
(g) to protect against any other hazards than that from which the harm has resulted.

c. Stachniewicz v. Mar-Cam Corp.

i. Plaintiff was involved in a bar room brawl and suffered


amnesia. He brought suit against the bar owner for negligence per se, as he violated a
statute that prohibited him from serving alcohol to “a person visibly intoxicated.”

ii. Courts must look to the type of harm and class of


persons in a statute, but also to the appropriateness of the standard it might set in
matters of civil litigation.

d. Impson v. Structural Metals, Inc., Tex. 1972

i. A truck attempted to pass a car within 100m of an


intersection, which was prohibited by a statute. The plaintiff claimed negligence per se
under the violation of the statute.

ii. § 288A. Excused Violations


(1) An excused violation of a legislative enactment or an administrative regulation is not
negligence.
(2) Unless the enactment or regulation is construed not to permit such excuse, its
violation is excused when
(a) the violation is reasonable because of the actor's incapacity;
(b) he neither knows nor should know of the occasion for compliance;
(c) he is unable after reasonable diligence or care to comply;
(d) he is confronted by an emergency not due to his own misconduct;
(e) compliance would involve a greater risk of harm to the actor or to others.

F. Proof of Negligence: Res Ipsa Loquitur

1. Elements that need to be present:

a. The accident which produced a person’s injury was one which


ordinarily does not happen unless someone was
negligent.

b. The instrumentality or agent which caused the accident was


under the exclusive control of the defendant, and

c. The circumstances indicated that the untoward event was not


caused or contributed to by any act or neglect on the
part of the injured person.

2. Basic Concept: Res Ipsa comes into play when there is no evidence as
to what caused an accident but it can reasonably be inferred that it was a breach of duty.

35
36

Prof comment: If you invoke this, it can get to the jury if there is no specific evidence of
what happened. Some jurisdictions say this a presumption, some say inference, some say
disappears altogether; Element (b) has been dropped in most jurisdictions because it
really isn’t fair, specifically regarding products liability cases. There can be too much
evidence to invoke res ipsa. Medical malpractice provides a bit of a different approach
because an expert is needed to testify when negligence may be inferred.

3. The Cases:

a. Eaton v. Eaton, N.J. 1990

i. Father sues his daughter for the wrongful death of his


wife. Daughter crashed a car and there is no evidence that anything other than daughter’s
negligence could have caused the accident (car flew off a cliff)

ii. Court sets forth elements of res ipsa here.

b. Ybarra v. Spangard, Cal. 1944

i. A guy goes under the knife and wakes up with a pain in a


non-related site that gets worse over time. Plaintiff cannot prove who did it, when, or
even how, but invokes res ipsa. Everyone is pretty much sued.

ii. Prof thinks this is the way of smoking out the guilty
party, “we know somebody knows.” Courts may have the go ahead to let a plaintiff
“sue em all” and let the jury find at least one of them that is most responsible.

c. Sullivan v. Crabtree, Tenn. Ct. App. 1953

i. A truck driver let Sullivan ride with him and ended up


getting into an unexplainable wreck which caused Sullivan’s death.

ii. There are degrees to which res ipsa may be invoked;


some warrant an inference of negligence that the jury may decide, another may require a
presumption of negligence if the defendant does not provide sufficient evidence to rebut,
and the other is a complete shift of the burden of proof to the defendant and requires that
he proof beyond a preponderance of the evidence that he was not negligent.

IV. Actual Causation

A. “But For” or substantial contribution (useful for indivisible harm) causation:


Did the defendant’s negligence actually cause the plaintiff’s harm?

36
37

1. The Cases:

a. Perkins v. Texas

i. Train is going faster than the speed limit and hits the
truck. It is argued that if the train had not been negligently speeding, then the harm
would not have occurred. Engineer testifies, however, that it would have occurred.

ii. When the plaintiff’s harm would have occurred even


if the defendant had not acted negligently, then the defendant’s negligence did not
legally cause the plaintiff’s harm.

b. Ford v. Trident Fisheries Co.

i. Guy was thrown overboard and no one heard or saw


the accident. When the crew members realized that
he disappeared, they attempted to rescue him.
When the crew finally realized he was gone, there
was a rescue attempt.
ii. Court said no negligence because there was no
causal connection.

c. Lyons v. Midnight Sun

i. Another speeding car wreck case. If the truck weren’t


speeding, would the harm have occurred.

d. Reynolds v. Texas Pacific Ry.

i. Large woman fell down stairs when she was hurried by


the railroad company into a stairwell that had no rail and was not lit. Defendant argues
that, even if the company were negligent for not lighting the stair, plaintiff might have
fallen anyway…the failure to light the stairs was not a cause of the harm

iii. Where the negligence of the defendant greatly


multiplies the chances of accident to its
occurrence, the mere possibility that it might
have happened without the negligence is not
sufficient to break the chain of cause and effect
between the negligence and the injury.

2. “What Would Have Happened?” Heeding Presumption

a. Failure to Warn

i. Obvious risk – no duty to warn.

37
38

ii. Not obvious but defendant was aware of the risk – no


duty to warn

iii. Not obvious and no warning label – may be a burden of


proof shift under the “heeding presumption,” that the ╥ is given credit for heeding a
warning if it had been presented; defendant then has to prove the ╥ would not have
heeded the warning.

B. Special Problems of Proof – the Quality of the Evidence

1. Professor’s take is that there are a ton of problems out there (birth
defects, cancer, etc.) that have some causal association with some things and not with
others. 1 in 50 people who smoke for over twenty years get lung cancer…this guy has it
and is suing. 1 in 50 of the people who get lung cancer got it because they smoked. How
does this guy prove that smoking caused him to get lung cancer, and that he isn’t just a
background case? Even with epidemiological studies, there’s not way to definitively
prove cause. It’s much easier with a marker disease (Mesophelioma only happens when
you are exposed to asbestos).

2. The Cases:

b. Daubert v. Merrell Dow Pharmaceuticals, Inc.

i. U.S. Supreme Court Case…Alleging birth defects from a


drug called Bendectin; plaintiffs introduce non-published expert testimony to
demonstrate the potential for the drug causing the problem. Court overrules the Frye
evidence test for the federal rules of evidence. The Frye test was that the evidence should
have been generally inadmissible unless the technique is generally accepted (aka peer
reviewed at least).

ii. Evidence has to be scientific (factors…tested, reviewed, rate of error, general


acceptance) and assist the tier of fact to understand or determine a fact in an issue
(relevant to the case). This is a flexible rule that has actually made it harder to get
evidence into court. Prof comment: There is a lot of time and effort that now goes into
knocking out the other side’s expert witnesses out. These principles will need to be
applied in a given case, or at least attempt to apply them. If you knock the other side out,
you get summary judgment.

c. Herskovits v. Group Health Cooperative of Puget Sound

i. Plaintiff alleged that Group Health negligently failed to diagnose Herskovits’ cancer
on his first visit to the hospital and proximately caused a 14% reduction in his chances of
survival. It is undisputed that he had less than a 50% chance of survival.

38
39

ii. Loss of chance, even if the defendant did not have greater than a 50% chance of
survival may be characterized as an injury for which the plaintiff can recover.

*The court says that once a plaintiff has demonstrated that the defendant’s acts or
omissions have increased the risk of harm to another such evidence furnishes a basis for
the jury to make a determination as to whether that increased risk was substantial in
bringing about harm.

iii. The minority of courts refuse to allow recovery for loss


of chance unless the plaintiff can establish causation under the traditional negligence
standard. The majority allow LOC claims to reach the jury even when the plaintiff
cannot prove the defendant was, more likely than not, the cause of the plaintiff’s harm.

v. Prof. comment, “you now may be able to recover for


less than 50% and the argument is that it’s unfair, why should they recover the entire
amount…but then, if you give only a percentage, why should you have to pay all if it’s
51%?”

B. When Two (or more) Negligent Actors Concurrently (or successively) Cause
the Plaintiff’s Harm

1. Joint and Several Liability: the doctrine applies where there’s more
than one tortfeasor and the damages they each cause are indivisible. It means that the
torfeasors are all jointly liable for any resulting judgment against them, and each of them
is also individually liable for the whole judgment (in case the plaintiff sues any one of
them instead of all of them).

a. applies when individuals acted independently but the harm is


indivisible (ex. two guys shooting a gun in the direction of the plaintiff and he’s hit with
one bullet).

b. applies when there is a concert of action (ex. drag racers and


one hits a kid, but are liable).

c. Prof. comment: “if you can figure out who is responsible for
what then no joint and several liability.”

d. “But for” test usually will not work in this case so gotta go with
the Restatement, Second, of Torts § 431-433 “an actor’s negligent conduct is a legal
cause of harm to another if…his conduct is a substantial factor in bringing about the
harm.”
2. The Cases:

a. Hill v. Edmonds

39
40

i. The owner of a tractor truck on a stormy night left it


parked without lights in the middle of a road where the car in which plaintiff was a
passenger collided with it from the rear, but the plaintiff had enough room to see the car
and avoid it. ***Marks the beginning of the phenomenon of indivisible harm.

ii. What if the street lights were off in the hill fact pattern
and the city was negligent, would the city also be responsible? Then we’d have three
joint tortfeasers. What if the car lights were off due to the negligence of the mechanic
and the lights contributed to the accident? Would that be four joint tortfeasers? Is there
any arbitrary limit on how many joint torfeasors you can have? No, you can have lots of
joint tortfeasers.

b. Kingston v. Chicago

i. Plaintiff’s property was destroyed by a large fire that was


the product of two smaller fires that combined north of plaintiff’s property. The
defendant, a railroad company, initiated a fire by sparks to the northeast of the plaintiff’s
property. The fire in the northwest was of unknown origin.

ii. When you can’t divvy up the damages because it’s


impossible, everybody that contributed to it is responsible for all of the damages.
The court does allude to a possible exception if the other tortfeasor had been mother
nature. – if the other perpetrator was mother nature than the fire might have happened
anyway, therefore no liability.

c. Summers v. Tice

i. Summers and the two defendants were bird hunting


together when defendants shot at a quail that flew in between Summers and themselves.
Summers was struck in the eye and the upper lip.

ii. In some circumstances, when plaintiff cannot prove


which defendant caused the injury, the plaintiff may join defendants – even without
a concert of action – and let them use their “superior knowledge” to apportion
damage.

*The basis for a decision like this is that the court does not want to send a person who has
been injured under no fault of their own empty handed. POLICY!!!!!

iii. Yes, we should still hold them both liable even though
one of them is completely innocent. Prof. gives hypo: guy shoots person in one eye and
other guy shoots in other eye, is there an indivisible injury? Possibly, blindness might be
the whole injury…depends on how you characterize the injury.

d. Sindell v. Abbott Laboratories

40
41

i. The plaintiff is the child of a women who took DES, a


drug intended to assist in the prevention of miscarriages. The plaintiff claimed to have
developed a bladder tumor as a result of the drug and sued 5 out of 200 manufacturers.

*In addition to concert of action, the plaintiff relies on two theories for her case:
1. alternative liability – when two or more defendants that inflicted harm and the plaintiff
cannot prove which inflicted it, the burden or proof may shift to each defendant to prove
he was not the one that inflicted the harm.
2. enterprise liability – an entire industry can be held liable for a product’s shortcomings
if the entire industry uses the same product, utilizing the same tests and strategies for
marketing of that product.
ii. Market Share Liability Theory - If the ╥ can’t prove
who manufactured the DES, the defendants may be responsible for their market share at
the time. The idea is that it will all even out over the long run. Over time, every
manufacturer will end up paying their fair share. This has applied to DES and pretty
much nothing else.

V. Proximate Cause - Breakfast Jurisprudence


A. Proximate cause, also known as legal cause, is mainly a question of policy.
Although each can reach the same conclusion, there are generally two approaches by the
courts:

1. Direct Cause: looks backward and determines if there are intervening


factors that should cut off responsibility. Popular in questions of Eggshell Skull Rule
theories, because it really isn’t foreseeable that the plaintiff has an eggshell skull.

2. Forseeability (also called the risk rule): looks forward at what was
foreseeable at the time of the activity. Something just needs to be generally foreseeable,
not specifically, unless the court is looking for a way to find no proximate cause that is.

*****In Nelson’s opinion this is always a policy question!

Generally, negligence that causes a delay to someone is not sufficient for recovery, must
be personal injury. Unforseeable plaintiff is also a snazzy cop out when needed.

B. The Hornbook
Some notes from E&E –
- The plaintiff can usually only recover for injuries the defendant should have
anticipated, or foreseen. If there is some foreseeable injury because of the
defendant’s action, he is liable for that strictly, and not any further injury that
results that is unforeseeable. (Wagon Mound, foreseeable oil spill would foul
the dock slips, but not catch fire – defendant liable for dock slips alone)
- If a particular type of injury is foreseeable, the defendant is liable for the
injury sustained, even though it might be more severe than anticipated.

41
42

- The cases distinguish unforeseeable consequences of a negligent act from


consequences that are foreseeable but take place in an unusual way.

C. The Cases:

1. Marshall v. Nugent

a. Oil truck ran a guy off the road in icy conditions. The guy was
hit by another vehicle when he was walking up a hill to warn oncoming traffic about the
oil truck’s presence in the middle of the road. Is the oil truck the proximate cause of the
guy’s injuries?

b. Proximate cause is a flexible doctrine. As a general rule, if the


defendant causes a delay, which results in a plaintiff’s injury, the defendant is not
liable. Also, if you negligently injure someone and the person is hurt on the way to
the hospital, in the hospital, etc. the defendant is liable for the damages.

*The court comments that foreseeablity is what determines if a situation is ongoing, such
as the negligence of the truck resulting in next accident, and a situation where everything
is stabilized and returned to normal.

c. What if other drivers got out and tried to help? If they were,
they wouldn’t be able to recover. What if a burglar jumped out of the bushes and shot
Marshall? What if you are in the vicinity of a prison and then there’s a sign up warning
you about this? Does that make it more foreseeable? What if an incoming vehicle
swerves to avoid the vehicle, flies off a cliff, and lands on a school (it is filled with
explosive material) blows up and destroys school. Is there proximate cause here?

2. McCahil v. New York

a. Skipped in class, support the “take your plaintiff as you find


him,” eggshell skull theory with regards to what can properly be foreseen.

3. In re Polemis and Furness, Withy and Co. – Direct Cause Approach

a. Asks the question, should the eggshell skull theory apply to


property? Dropping a plank on accident caused a spark which blew up the entire ship.
The defendant could not have anticipated that dropping the plank would have caused an
explosion.

b. As long as the harm is directly traceable to the defendant’s


negligent act (meaning no intervening cause), the plaintiff may recover so long as it
is foreseeable by a reasonable person that the act could cause SOME damage, no
need to foresee what kind or to what extent.

4. Wagon Mound #1, brought by the wharf owners

42
43

a. Defendants owned a ship that spilt a type of oil into the ocean
that was supposed to be basically non-flammable. The wharf
repair crew was using acetylene torches and a spark hit the
water, ignited the oil, and blew up the wharf; the plaintiff’s
don’t want a forseeability approach because they
b. should have foreseen it also and would be contributorily
negligent.

*The Wagon Mound case in essence overrules Polemis, stating that there must be
foreseeability that the defendant’s action would cause damage.

5. Wagon Mound #2, brought by the ship owners at the wharf

a. The court can use utility to squirm…they use a cricket case as


precedent and classify the activity as playing cricket; in the wagon mound case they
classify the activity as spilling oil (why not transporting oil, there’s plenty of utility in
that!?). Under this logic, the cricket activity should have been classified as hitting
someone with a baseball.

b. Forseeability approach is popular for masking another agenda,


very malleable. If the court doesn’t want to find a defendant liable, they get very specific
(couldn’t foresee blowing up the ship with a spark) and if they do, they are very general
(could have foreseen some harm, doesn’t matter what kind or to what extent).

6. Palsgraf v. Long Island R.R.

a. R.R. worker helped push a guy up on a train when he was trying


to jump on while it was moving and, as a result, he dropped a package containing
fireworks that managed to cause some damage and knock over scales that injured the
woman. The suit was trying to hold the railroad worker liable.

b. The court holds that Palsgraf is an unforeseeable plaintiff,


and that the RR breached no duty it owed to her.

c. (Dissent) factors of proximate cause:

 must be, at the least, something without which the event would not happen

 there was a natural and continuous sequence between cause and effect

 Was the one a substantial factor in producing the other?

 Was there a direct connection between them, without too many intervening
causes?

43
44

 Is the effect of cause on result not too attentuated?

 Is the cause likely, in the usual judgment of mankind, to produce the result?

 Or, by the exercise of prudent foresight, could the result be foreseen?

 Is the result too remote from the cause - for the greater the distance either in time
or space, the more surely do other causes intervene to affect the result

*Prof. Comment – “The concept of foreseeablity is more of a tool to shape a decision the
court has already or seeks to make.”

D. Superceding Causes

1. Derdarian v. Felix Contracting Corp.

i. Construction worker was hit by an epileptic guy while he was


working just off the side of the road. The contractors didn’t have a proper barrier in
place so the question become whether the contractor’s negligence was the proximate
cause of the plaintiff’s injury.

ii. An intervening act may not serve as a superseding cause,


and relieve an actor of responsibility, where the risk of the intervening act occurring
is the very same risk that renders the actor negligent.

2. Watson v. Kentucky & Indiana Bridge & R.R.


i. The defendant allowed gasoline to escape from a railroad tank
car and flow into the streets, filing gutters and standing in pools. The plaintiff was
injured when a third party threw a match into a pool of the spilled gasoline and it
exploded. Plaintiff claimed the defendant is liable for negligently spilling the gasoline
regardless of the actions by the third party.

ii. Court is saying that it’s unforeseeable as a matter of law that


a third party will commit a criminal act.

3. Fuller v. Preis

i. Dr. was hit in an accident and suffered no apparent injury at the


time, but later decided to kill himself.

ii. Suicide is not a superceding cause as a matter of law; it does


not bar recovery for loss of life. There is an argument that it may be contributory
negligence.

4. Wagner v. International Railway

44
45

i. Plaintiff fell from a bridge looking for his brother who was
negligently thrown from the railroad.

ii. The rescue doctrine – defendant owes a duty to the rescuer;


If the rescue is not wanton/reckless, then the defendant is liable (even third
party). One exception has been held that there is no liability if the rescuer has not
deterred anyone else from rescuing. Also, the rescuer must be attempting to actually
rescue.

*The defendant need not foresee that a rescuer would come.

iii. From the Cardozo perspective, this is carving out an exception


to his unforeseeable plaintiff doctrine, that rescuers are still allowed to recover.

E. “Playing the Duty Card”

1. Hamilton v. Beretta U.S.A. Corp.

i. A huge class action from victims of hand guns against


manufacturers for negligence.

ii. As a general rule, a defendant is not liable for criminal acts of a


third party; Special relationship exception: there is a relationship either between ▲ and
third person tortfeasor that encompasses defendant’s actual control of the third person’s
actions, or between defendant and plaintiff that requires defendant to protect plaintiff
form the conduct of others. (defendant controls a third party or owes a duty to protect
plaintiff).

VI. Limited Duty Rules


A. Limitations under the Duty to Rescue:
1. General Principles: Under common law there is no duty to rescue but
there are some exceptions: special relationships running between a common carrier and
its passengers, a school and its students, and an employer and its injured employee. Also,
once someone has begun a rescue, they have a duty to go about it in a reasonable manner.
Further, if you innocently or negligently create the risk, you are under a duty of care to
prevent the risk from taking effect.

2. The Cases:

a. Yania v. Bigan

i. A guys basically gets dared into jumping 20ft into a pool


of water and drowns. The defendant just watched and did not help.

45
46

ii. There is generally no duty to rescue. The court holds


that the defendant was an adult in full possession of his mental capacities.

iii. § 322 of the Restatement, Second, of Torts:

If the actor knows or has reason to know that by his conduct whether tortuous or
innocent, he has caused such bodily harm to another as to make him helpless and
in danger or future harm, the actor is under a duty to exercise reasonable care to
prevent such future harm. Professor makes it a point to argue that there may be a duty
here if you use this restatement and argue “cause.”

b. Eerie R. Co. v. Stewart

i. Railroad company normally has a watchman out. One


night he wasn’t out and a guy, who knew him to be there when danger was present and
relied on him, moved out onto the tracks and was killed.

ii. If the defendant has caused the plaintiff to rely on a


standard of due care it must make a reasonable effort to inform that the plaintiff
when the care is being discontinued. If they did not, negligence as a matter of law.

c. L.S. Ayres & Co. v. Hicks

i. Hicks, a six year old boy, was shopping at the


defendant’s department store when he slipped on the escalator. The fall caused Hicks to
catch his fingers in the escalator.

ii. there may be a legal obligation to take positive or


affirmative steps to effect the rescue of a person who is helpless and in a situation of
peril, when the one proceeded against is a master or an invitor or when the injury resulted
from use of an instrumentality under the control of the defendant. Such an obligation
may exist although the accident or original injury was caused by the negligence of the
plaintiff or through that of a third person and without any fault on the part of the
defendant.

iii. Prof. Comment: “The thing to take away is that it


doesn’t matter whether the defendant was negligent or not. The defendant is the only one
available to assist the plaintiff and the defendant’s failure.” Special relationship may
give rise to a duty.

d. J.S. and M.S. v. R.T.H

i. A wife might have known that her husband was


molesting children over a period of years. The issue was whether she had a duty to act on
that knowledge or not.

46
47

ii. When a spouse has actual knowledge or special reason


to know of the likelihood of his or her spouse in engaging in sexually abusive behavior
against someone, the spouse has a duty to take reasonable steps to prevent or warn of the
harm. A person who has a measure of control over another has a duty to attempt to
control them in certain situations.

e. Tarasoff v. Regents of University of California

i. Case where the psychologist heard a guy say that he was


going to kill someone. He told the cops and the cops determined that he was ok. Then he
killed the person.

ii. Special relationship – an exception to the general rule


that a defendant owes no duty to warn of is control over the conduct of a third
person. This relationship existed between the psychiatrist and the victim.

iii. Professor comment: Remember in Tarasof the baseline


rule is that you don’t have a duty to prevent a third party from preventing harm to
another or prevent them from committing suicide. This is the g/r.

B. Limitations on Recovery for Pure Economic Loss

1. General Principles: As a general (bright line) rule, you cannot recover


for economic damages alone but can recover for them when there has been some personal
or property injury. Quasi property interests might be enough, like a fishing company that
lost money because a chemical spill killed off the fish supply. You need to also look to
whether the economic damages result from the damage to the person or property.

2. The Cases

a. State of Louisiana ex rel Guste v. M/V Testbank

i. Boats collided and caused a navigation halt on the MS


river. As a result, the fishing, shrimping, and related industries suffered considerably
economic damages.

ii. court upholds the bright line rule (no recovery for pure
economic loss) but there are concurring and dissenting opinions that lay out the
arguments against it.

b. J’Aire Corp. v. Gregory

i. restaurant owner brought suit for economic losses


incurred by a contractor that failed to completely timely repairs.

ii. J’Aire Factors:

47
48

(1) the extent to which the transaction was intended to affect the plaintiff
(2) the forseeability of harm to the plaintiff
(3) the degree of certainty that the plaintiff suffered injury
(4) the closeness of the connection between the defendant’s conduct and the injury
suffered.
(5) the moral blame attached the to the defendant’s conduct
(6) the policy of preventing future harm

c. People Express Airlines, Inc. v. Consolidated Rail Corp.

i. The defendants’ negligence resulted in leaking chemicals


which caused the airlines to close down and suffer loss of business for a few days.

ii. This court holds that economic losses are recoverable


without damage to property or persons if the plaintiff is particularly foreseeable.

C. Limitations on Recovery for Emotional Distress (NIED)

1. General Principles: Courts have tough standards for considering


“emotional distress” as damages under negligence.
Impact rule – there can be no recovery for distress without actual impact (at one time
was the clear weight of authority).
Zone of Danger rule – recovery without impact was allowed but needed to be in the
zone of danger (eventually replaced impact rule).
While impact and zone of danger applies to bystanders, direct victims (i.e. those that are
emotionally injured by negligent notification of death, negligent handling of corpses,
negligent child birth cases [something happens to the baby and the mother either
witnesses it or finds out soon thereafter], negligent handling of genetic material, cancer
phobia cases, AIDS phobia, etc.) are usually allowed to recover. Liability is generally
imposed for the mishandling of a loved one’s corpse.

2. The Cases:

a. Daley v. LaCroix

i. A vehicle struck a utility pole which caused an


explosion that injured the plaintiff’s property and “emotionally distressed them.”

ii. Court overrules the impact rule. Where a definite and


physical injury is produced as a result of emotional distress proximately caused by
the defendant’s negligent conduct, the plaintiff may recover even though he did not
suffer any physical impact.

b. Thing v. La Chusa

48
49

i. A woman came upon a crash seen and saw that her son
was badly injured. She wasn’t impacted or in the zone of danger but was emotionally
distressed.

ii. Under Dillon, elements for NIED:


(1) closely related to the injury victim
(2) present at the scene of the injury-producing event at
the time it occurs and is then aware that it is causing injury tot the victim
(3) as a result suffers emotional distress beyond which
would be anticipated in a disinterested witness.

iii. “These are bystander recovery cases…my son was


watching TV when the challenger blew up and he was upset for a long time…should he
have been able to recover under NIED?”

*Under Dillion the elements for recovery are semi-felixble. Under Thing they become
strict.

D. Harm to Unborn Children

1. General Principles:

a. Wrongful Conception: The doctor botched the sterilization and


a woman gets pregnant. If the child is born, the courts will allow recovery for medical
expenses from birth but not child rearing expenses unless the child is born with a
handicap.

b. Wrongful Birth: The doctor failed to warn of some defect, and


the parents would have chosen to have an abortion if they had known. Roe v. Wade
kicked this off, recovery for negligently “taking away the right to choose.” Some courts
will allow recovery, some will not, some allow for general damages, some do not. A
parent brings this claim.

c. Wrongful Life: This is a child’s claim. Courts are reluctant to


consider this complaint because it is almost like arguing over whether someone would
have been better off not born at all. This occurs when a child has a defect or disability.

2. The Cases:

a. Werling v. Sandy

i. The plaintiff brought a complaint alleging that the


doctors were responsible for their child’s stillborn birth.

ii. Action for the death of a fetus in utero or a stillborn


infant is allowable as long as the injury occurred when the fetus was viable.

49
50

b. Procanik by Procanik v. Cillo

i. An infant brings suit for negligence against doctors who


failed to warn his mother that her measles during first trimester of pregnancy would result
in his disabilities. Mother claims she would have terminated the baby. As a result,
plaintiff was born with congenital rubella syndrome.

ii. Parents were allowed to recover special damages for


extraordinary medical expenses until the kid reached the age of majority and then he
would receive the money.

*Here the court reiterates its reluctance to allow recovery for impaired childhood and
basically say that it would have been better for the kid to have never been born.

VII. Owners and Occupiers of Land


A. Duties Owed to Entrants on the Land

1. Trespassers: If the trespasser is unknown, there is a duty only to


refrain from wanton or reckless conduct. If the presence is known – or should be
known – the land owner is under a duty of care to warn of hidden dangers or use
reasonable care. Constant trespassers are afforded a higher duty of care, because the land
owner has given implied consent (unless they convert them to full trespasser with
something like a “no trespassers” sign).

a. Restatement Second of Torts, Children Trespassers; attractive


nuisance doctrine:

§ 339. Artificial Conditions Highly Dangerous To Trespassing Children

A possessor of land is subject to liability for physical harm to children trespassing


thereon caused by an artificial condition upon the land if
(a) the place where the condition exists is one upon which the possessor knows
or has reason to know that children are likely to trespass, and
(b) the condition is one of which the possessor knows or has reason to know and
which he realizes or should realize will involve an unreasonable risk of death or
serious bodily harm to such children, and
(c) the children because of their youth do not discover the condition or realize the
risk involved in intermeddling with it or in coming within the area made dangerous
by it, and
(d) the utility to the possessor of maintaining the condition and the burden of
eliminating the danger are slight as compared with the risk to children involved, and
(e) the possessor fails to exercise reasonable care to eliminate the danger or
otherwise to protect the children.

2. Licensees: persons who are on the land with the express or implied
consent of the owner but are there for their own purpose, i.e. social guests, entrants that

50
51

use short cuts, distribute advertising leaflets, come to borrow tools, or solicit charitable
contributions. Land owners generally have a duty to conduct activities on the land in
a reasonable manner to warn of hidden dangers (natural or artificial) known to
him. There is no duty to inspect.

3. Invitees: Prof. comment, “there for the economic interest of the


owner.” The full reasonable care standard applies.
Restatement Second of Torts § 332 recognizes two categories of invitees:
(1) persons who are invited to come on the land for a purpose connected with the
business dealings of the possessor and
(2) persons who come on the land as a member of the public for a purpose for which the
land is held open to the public. There are public invitees (person entering a public library
to borrow a book, a person entering a drugstore to use the public telephone) and business
visitors (store customers, building constructors). According to the horn book, a land
occupier owes invitees a duty to use reasonable care to inspect and discover the
presence of any dangerous natural or artificial conditions or activities and to exercise
due care to warn invitees of such dangers or make the conditions or activities safe.

4. Other general principles: The statuses flip back and forth, one minute
might be a licensee until they break out the Tupperware idea and then they become an
invitee. The purpose for being in a public place matters (sitting in the parking lot to
socialize is not being there for invitee purposes) as does a failure to take action against
consistent trespassers. Firefighters Rule – can’t tax twice in many states, fire fighters
aren’t able to recover for injury on premises as a matter of public policies unless it is
negligence other than that which causes the need for intervention by the plaintiff.
Recreational Use statutes often let people who open their land up to the public for no
money get away will less liability or even no liability.

51
52

52
53

5. The Cases:

a. Gladon v. Greater Cleveland Regional Transit Authority

i. Guy was a passenger on a train and was beaten and


doesn’t know if he was thrown into or purposefully walked into the section of the tracks
and was hit by a train. He was severely injured.

ii. The visitor has the status of an invitee only while he


is on part of the land to which his invitation extends. In this case, he was not an
invitee while on the tracks. The status of an accidental trespasser is still that of a
trespasser.

b. Rowland v. Christian

i. A social guest was at a woman’s apartment and cut her


hand on the defendant’s faucet. The defendant knew the faucet was in a bad condition.
The duty to licensees in California at the time was to refrain from willful or wanton
misconduct.

ii. The California duty to a licensee was to refrain from


wanton or willful injury with two exceptions: (1) active conduct of negligence (2) hidden
trap (spring gun type of thing). California could have stretched this case under the
second exception or broadened the duty to a licensee. California court gets rid of
categories altogether, most courts don’t follow. Lays out the “Rowland factors” on p.
375.

*The California court in this case was the first to get rid of the categories, stating that “a
man’s life and limb is not worth less because he is not on a property for a business
purpose.”***POLICY

c. Carter v. Kinney

i. Guy slips on ice while going to a bible study and tries to


get status as an invitee.

ii. Court says nope. Prof. comment, “In this situation, does
the licensee category work well? So maybe this is a case where the categories work sort
of an injustice.”

*The court reiterates in this case that if an owner of land “throws open its doors to the
public” everyone that enters is an invitee.

53
54

B. Duties to Those Outside the Premises

1. General Principles: Under common law there are two categories of


liability: natural (no liability) or artificial conditions (reasonable care standard) and
activity. The book gives an example of an artificial condition coming from the land as a
baseball being hit and impacting someone outside the park versus natural conditions like
snow melting and flooding someone else’s yard, etc. There are tricky areas to this rule
when trees fall from land onto roads or sidewalks. Many courts make a distinction
between rural and urban trees.

2. The Cases:

a. Taylor v. Olson

i. Plaintiff’s car ran into a tree that fell across the road into
the enjoining highway. The tree was on the defendant’s land

ii. Discusses categories of duty between rural (no opinion


on a duty) and urban (due care) areas and abandons the categories and says there was a
duty to inspect and the jury should decide whether that duty was breached.

C. Duties Owed by Lessors

1. General Principles: Majority of courts recognize this duty with these


exceptions: Unless the vendor knows of the dangerous condition and conceals it from the
purchaser, he is not liable for injuries that occur after the vendee has taken possession.
Furthermore, once the vendee discovers the dangerous condition and has the opportunity
to correct it, the vendor’s obligation comes to an end.
a. Exceptions:

i. A hidden danger in the premises of which the landlord


but not the tenant is aware.

ii. Premises leased for public use.

iii. Premises retained under the landlord’s control, such as


common stairways.

iv. Premises negligently repaired by the landlord

2. The Cases:

a. Sargent v. Ross

i. Plaintiff’s four year old daughter fell to her death down


defendant’s stairwell. The landlord was sued for negligent construction and maintenance

54
55

of the stairway which was added to the building by the defendant about eight years before
the accident.

*The court basically seeks to shift the question from “who had control here?” to “did
both parties exercise due care under the circumstances?”

ii. Scrapped the categories. Prof comments, “if the


landlord repairs and someone is hurt than it meets an exception and is liable so there are
no incentives for the landlord to repair. The renters really don’t have an incentive
either, b/c it’s not their property.”

D. Premises Liability: Securing Against Crime

1. General Principles: Courts are reluctant to impose a duty of reasonable


care to protect against criminal conduct, but there are exceptions. Some courts do find
that business owners have a duty to implement reasonable measures to protect their
patrons from criminal acts when those acts are foreseeable. The majority of courts do
impose liability on lessors for failing to maintain reasonable security.

2. The Cases:

a. Posecai v. Wal-Mart Stores, Inc.

i. Woman was mugged in the Sam’s store and had $19,000


of jewelry on her. The plaintiff alleged that Sam’s failed to exercise reasonable care to
prevent crime in the parking lot and sued for negligence.

ii. Looks closely at forseeability and a number of tests that


were available to the court, adopting the “balancing test” and determining that crime was
rare in the parking lot, so it was not foreseeable enough to put a security guard in the
parking lot.

VIII. Affirmative Defenses


A. Defenses Based on Plaintiff’s Conduct

1. Contributory Negligence: (still around in a few states) conduct on the


part of a plaintiff that is a contributing cause to her own injuries, and that falls below the
standard to which she is required to conform for her own protection.

*Basically does not allow recovery, not matter how small plaintiff’s fault was. The court
thought this was a little too harsh, so it instituted exceptions:

a. Exceptions:

55
56

i. Last Clear Chance, if plaintiff was negligent but was in


a position of peril unable to extricate herself from danger and the defendant discovered
the plaintiff’s peril, then plaintiff’s contributory negligence was not a bar. In other
words, the plaintiff’s negligence contributed to the injury but the defendant had the last
chance to avoid the injury and, therefore, plaintiff was helpless.

ii. Defendant’s conduct constituted an intentional tort or


could be characterized as reckless or wanton, contributory negligence is not a defense.

b. The Cases:

i. Butterfield v. Forrester

aa. Plaintiff was injured by a pole that defendant


put up in the middle of the street. He was thrown from his horse and severely. He was
speeding on his horse and should have seen the pole from 100 yards away.

bb. The court refused to allow the plaintiff to


recover and set forth the doctrine of contributory negligence.

2. Comparative Negligence %: (now adopted by virtually all states)


rejects the notion that contributory negligence is always a complete bar to recovery by the
plaintiff. Instead, the approach attempts to individualize accident recoveries by placing
the economic ‘sting” on the parties in proportion to their fault.

a. Pure Comparative: is 90% at fault, then the he can still


recover for 10% of damages.

c. Modified Comparative: There are two types of jurisdictions


for this:
a. 49% - plaintiff’s amount of fault can’t be over
b. 50% - plaintiff’s amount of fault can’t be over
c. If there are multiple defendants, the vast majority of
courts allow a the defendants’ fault percentages to be
aggregated when being compared to the plaintiff’s.
i. D1 – 10%, D2 25%, D3 25% - P40%
ii. Above is really – P40%, Ds’ 60%

c. The Cases:

i. McIntyre v. Balenstine

a. Plaintiff was intoxicated when he was struck in


his truck by the defendant who had also been drinking and was driving a tractor over the
speed. This is a case showing the movement from contributory negligence to
comparative negligence.

56
57

ii. Alami v. Volkswagen of American Inc.

a. A drunk guy rammed into a pole and died. The


widow brought a products liability suit alleging that the vehicle was defected and, if it
weren’t, the her husband would have lived. The issue became whether his criminal
conduct – driving while intoxicated – was a bar to recovery.

b. Courts may not bar recovery if the plaintiff was


injured while committing a crime if it is a matter of public policy.

d. The effect of the shift on other law: Most court have abandoned
last clear chance. Some have abandoned joint and several liability. Intentional torts and
reckless conduct become circumstances for the court to assess under comparative fault.
A problem arises under modified comparative fault when there are multiple defendants.
A small minority of states, under modified comparative fault, will deny recovery to a
plaintiff that is more at fault than any one single defendant but less than the aggregate
fault of all defendants. Most will allow it.

3. Assumption of the Risk

a. Express assumption of the risk: (still alive after comparative


negligence) This usually takes the form of exculpatory clauses in a contract, but courts
will apply it to sports at times. An exculpatory clause may or may not be enforced by a
court if the harmful conduct is negligent (contract law play a part, ex. adhesion
contracts). However, intentional torts or reckless or wanton conduct is NEVER
enforceable.

i. Jorst v. D’Ambrosio Bros. Investment Co.

aa. Plaintiffs signed a release for riding on a horse


ranch and end up hitting a PVC pipe on the track. They sued, and the court considered
whether the contract was enforceable.

bb. For a release to be enforceable, CA courts


identified three elements;
1. the release must have been clear and unambiguous;
2. the act of negligence that caused the injury must have been reasonably related
to the object or purpose for which the release is given;
3. the release must not be contrary to public policy.

ii. Tunkl v. Regents of University of California

57
58

aa. Note case, professor comment, “This is a


landmark case (significant decision) which involved the patient that was in a hospital
and signed a release. As long as the hospital was negligent in the selection of employees
the hospital was not liable (contract). The court set forth some factors but primarily
looked at the fact that it was an adhesion contract, take it or leave it. What about
arbitration agreements? If you wanna come here you gotta agree to arbitration. It can
happen but doctors hardly lose in AL so why would they.”

b. Implied Assumption of the Risk: (pretty much disappears


after comparative negligence doctrine) absent a contract, there may be implied
assumption of the risk if the plaintiff, by conduct, can be held to have voluntarily
assumed the particular risk involved.

i. Primary is a duty analysis, pretty much. (guy is in the


bleachers and gets hit by a ball, another way to say this is NO DUTY…comparative
negligence has no effect at all here; we should not really even be talking about this,
because it’s a duty analysis).

ii. Secondary is a comparative negligence analysis, pretty


much…in other words, there was a breach of a duty but the plaintiff’s confrontation of a
risk was negligent. (╥ voluntarily confronts a known risk, what about comparative
damages, does secondary implied assumption of the risk survive? Often it does not
because this doctrine is basically subsumed under comparative negligence, should no
longer be a complete bar to recovery)

iii. Blackburn v. Dorta

aa. No facts. Basically looks at home many courts


approach assumption of the risk…we don’t have to call it that anymore, it is just merged
into the concept of comparative negligence when the court is assessing whether the
plaintiff was reasonable in her conduct.

iv. Turcotte v. Fell

aa. Professional jockey injured during a race. The


╥ charged foul riding and negligent maintenance of the premises along with several other
things.

bb. General rule as to co-participants in sports:


their conduct has to be something more than negligence. Prof comment, “the basic
question is when should that no duty under assumption of the risk concept bar ALL
recovery rather than reduce recovery.”

4. Avoidable consequences: The plaintiff has a duty to mitigate the


damages – the focus is after the injury. An injured party must act reasonably to minimize
her loss or injury, and where the damages are unnecessarily aggravated or increased

58
59

through her failure to do so, the additional damages are NOT recoverable. DO NOT
CONFUSE THIS WITH CONTRIBUTORY NEGLIGENCE. This affects only the
damages and this failure to take reasonable measures in no way contributed to the
accident, ex. is not wearing a seatbelt.

a. Bryant v. Calantone

i. A guy had a heart murmur and was told to take a certain


amount of medicine before he had any dental work done. He told the doctor that he was
supposed the take the antibiotics weeks before the dental work and doctor said he didn’t
need it and simply gave him a pill. The guy was fine until he suffered severely later on.
The question became whether the guy failed to take reasonable precautions by not asking
for medication or a prescription afterwards.

ii. Prof comment, “the doctrine of avoidable consequences


should have been charged to the jury. The doctrine proceeds on the theory that a
plaintiff who has suffered an injury as the proximate result of a tort cannot recover for
any position of the harm that by the exercise of ordinary care he could have avoided.”

iii. Potential test material – comparative negligence vs.


avoidable consequences and damages in a modified comparative negligence jurisdiction.
The key is to focus on “post treatment” for avoidable consequences.

B. Non-Conduct Based Defenses

1. Immunities

a. Most interfamily immunities have been dropped with two


exceptions preventing a child from suing his or her parent: (1) where the negligence
involved parental discipline over the child (2) where the negligence involved exercise of
normal parental discretion over such matters as providing food, housing, and medical
services.

b. Charitable Immunities: (some rejected, some still around, some


around with a cap on recovery) reasons that support charitable immunities: (1) tort
recoveries would invade “trust fund” (2) tort claims would put charities at risk; however,
insurance has put somewhat of a cap on that.

c. Governmental immunity: Most governments have limited


immunities still. Federal Tort Claims Act (FTCA) abolished tort immunity against the
federal government with some exceptions: G/R the federal government is not liable for
intentional torts unless it is committed by a law enforcement officer. The
discretionary function exception is made to prevent the government for being sued for
policy reasons. The Feres doctrine (landmark Supreme Court military case) prohibits an
active duty military service member from suing the government from pretty much
anything (incident to service).

59
60

i. Tippett v. United States – Moose attacked a group of


snow-mobile folks and they sued, but the government contended that it was immune
under the Federal Tort Claims Act “discretionary function” exception. People have to
have SOME choice/discretion, almost always…look at the malpractice case mentioned in
the notes. How does it work when you have a physician treating a patient or doing
surgery? Physicians make choices. We will hold the government liable for malpractice
under the FTCA….the Gonzalez act says that when you sue a government physician the
US is also a party. That is not deemed to be a discretionary function. Case law is pretty
clear that medical malpractice does NOT come under this exception.

d. Municipality immunity – Riss v. City of New York: crazy case


where the woman sued the cops because they failed to protect her from her crazy stalker.
She ended up having lye sprayed in her eyes and nearly going blind. This case just
demonstrates arguments on both sides for why the state should be immune.

2. Statutes of Limitation may prevent suit from being brought.

3. Statues of Repose limit liability within a number of years for specific


groups (architects, engineers, manufacturers, etc.).

IX. Joint Tortfeasors

A. Comparative negligence and uniform tort acts have had drastic impacts on
joint and several liability in the several states and there are many different ways of
apportioning damages.

B. Actors may be jointly liable when: they are acting in concert, liability is
opposed by operation of law (doctrine of respondent superior), and when there has been
an indivisible injury occur.

X. Strict Liability (no fault necessary)

A. Possession of Animals

a. General Rules

i. Livestock – typically strict liability standard

ii. Domestic Animal - typically negligence standard unless the


following strict liability elements are met. Note, however, that the focus is on the
individual animal and not the class of animals such as dog or even rottweiler. Also, the
damage must be related to the known vicious propensity:

60
61

aa. The defendant kept the animal

bb. The animal had vicious propensities (every dog entitled


to one bit idea)

cc. The defendant knew or had reason to know (scienter)

iii. Wild Animal – typically strict liability standard as long as the


harm that occurred is related to the animals dangerous propensities.

b. Sandy v. Bushey

i. The plaintiff was kicked by the defendant’s horse and injured.

ii. This case addresses whether contributory negligence can be


established a defense to strict liability and the court determines that it may not, because a
standard higher than negligence must be established, BUT Nelson says, today,
comparative negligence or even a defense of assumption of the risk may be used.

B. Abnormally Dangerous Activities

a. General Principles: One who maintains an abnormally dangerous


condition or activity on his premises or engages in an activity that presents an
unavoidable risk of harm to the person or property of others may be liable for the harm
caused even if the defendant has exercises reasonable care to prevent the harm. The issue
is determining what constitutes an abnormally dangerous activity.

b. Restatement § 519

i. One who carries on an abnormally dangerous activity is


subject to liability for harm to the person, land,or chattels of another resulting from
the activity although he has exercised the utmost care to prevent the harm.

ii. Strict liability applied only to the kind of harm, the


possibility of which makes the activity abnormally dangerous.

c. Restatement factors to consider when determining if an activity is


abnormally dangerous (softening of strict liability):

• Whether the activity involves a high degree of risk of harm


• The gravity of that risk
• Whether the risk can be eliminated by the exercise of reasonable care
• Whether the activity is a matter of common usage
• Whether the activity is appropriate to the place where it is being carried on; and
• The value of the activity to the community

61
62

d. Extras: Doesn’t have to be commercial, can be any activity. Risk not


eliminated by due care. Heart of the test. Also, common usage. Cars aren’t
abnormally dangerous because of their common use. But if operating a tank,
different. Aviation used to be an abnormally dangerous activity. Now, if
plane falls on house, then strict liability. Or, if guy is doing stunts in a plane,
strict liability – probably in a car too, for that matter. Assumption of the risk
may be a defense: any voluntary encountering of a known risk may prevent
the plaintiff’s recovery.

XI. Trespass to Land and Nuisance


A. General Principles: the tort of nuisance protects one’s right to the use and
enjoyment of property. Unlike trespass, which protects against physical invasion of
property by another, the tort of nuisance protects against intangible invasion of one’s
land. It is fairly flexible. The distinction between nuisance and trespass is very
important: trespass involves a physical invasion of plaintiff’s land while nuisance
protects the plaintiff against interference with the use or enjoyment of the land.
There are situations where BOTH occur at the same time.

B. Private Nuisance v. Public Nuisance:

1. A private nuisance is a non-trespassory interference with the plaintiff’s


interest in the use or enjoyment of her property the results in substantial and
unreasonable harm to plaintiff’s interest in the use and enjoyment of the land.

a. Substantial means that a reasonable person would be offended


by it, not just a grouchy neighbor.

b. Unreasonable means that, taking all the factors into


consideration, the harm done by the interference outweighs justifications for the
defendant’s conduct.

2. A public nuisance is an act by a defendant that obstructs or causes


inconvenience or damage to the public in the exercise of rights common to all, or in the
enjoyment or use of common property.

3. A plaintiff usually cannot bring an action for public nuisance unless he


or she suffered an injury “peculiar in kind” apart from that common to the public
(e.g. an obstruction on a public road is a public nuisance, but it is a private nuisance as to
╥ it if also blocks her driveway). A private Nuisance plaintiff must have possessory
interest in the land meaning he or she is either in actual possession or has the right to
immediate possession.

Elements of a private nuisance:


1. a basis for liability
2. significant harm

62
63

3. an unreasonable invasion of the plaintiff’s land

C. Defendant’s Conduct

1. Restatement § 822

Basis of liability may be:


(a) unintentional and unreasonable
(b) unintentional and otherwise actionable under the rules controlling liability
for negligent or reckless conduct, or for abnormally dangerous conditions or
activities.

2. Basically, liability can fall under an intentional tort, negligence, or even


strict liability standard. Most nuisances are intentional because the plaintiff has
complained to the defendant and they’ve ignored the complaints.

63
64

64
65

XIII. DAMAGES
A. Compensatory Damages
B. E&E notes
a. Elements of Compensatory Damges
i. Medical expenses –past, present and future medical bills are
recoverable
ii. Lost Earnings and Earning Capacity – the jury works to determine
what loss of capacity is, especially in a situation where the person
is not in the career they were always going to be doing.
iii. Pain and Suffering – a catch-all term that in reality covers any
subjective reaction to an accident
iv. Loss of Enjoyment of Life – self explanatory?
C. Case notes
a. Primary instrument of recover in torts cases. Aim is to “restore plaintiff to
her pre-injury condition by paying an amount equal to the value of the
interests that the defendant has diminished or destroyed.”
b. Cases
i. Anderson v. Sears, Roebuck & Co –
1. A sears heater blows up and burns down the house injuring
a mom, a dad, and their baby girl (the latter being injured
very badly and permanently).
2. The court looks at the amount of compensatory damages to
decide if they are excessive. The court applies the
“maximum recover rule” which states that the trial
judge will determine whether a jury verdict exceeds the
maximum amount which a jury could reasonable find
and, if it does, the trial judge may reduce the amount.

ii. Richardson v. Chapman


1. Two women are involved in a collision. One is paralyzed
and awarded a mother-load in awards. As in last case, court
looks at this to determine whether this amount is too much.
2. The court holds that an award of damages is excessive if
it is:
a. Outside fair and reasonable range compensation
b. Results from passion or prejudice
c. Shocks the judicial conscience

c. Measuring losses
i. The court will sometimes modify awards

65
66

1. Either they will calculate future inflation in economy and


raise the award or
2. They will reduce the award to a value that reflects the
ability of the plaintiff to invest the award
ii. In determining future loss of wages for a person that has yet to
start a career, if the court has good reason to believe a person was
going to pursue a particular career (person injured while in law
school or med school) then the court will factor in the average pay
for that career specifically
iii. Compensatory damages are not taxed as income, even if they
represent lost waged

d. Non-economic losses (pain and suffering etc.)


i. Cases
1. McDougald v. Garber
a. Lady went into permanent comatose while
delivering baby due to doctor’s malpractice
b. The court was looking at the rewarding for loss of
enjoyment to a comatose person. The court
decided for an award for loss of enjoyment of
life, there must be some cognitive awareness
prerequisite.

e. Measuring economic losses


i. Cases
1. Coyne v. Campbell
a. Doctor is injured in car wreck and has injuries in
excess of 2,000. He got his doctor friends to fix him
for free, but still tried to get the money from the
defendant.
b. The court says here that the doctor can’t recover
because it would be, in effect the defendant paying
the value of a gift. COYNE IS A MINORITY
POSITION.

2. Most courts use the Collateral Source Rule, which says


evidence that a plaintiff received some sort of
compensation for his injuries from a third party, or
anyone besides the defendant, is inadmissible in court
a. The idea here is not to give the plaintiff double the
amount of money, but to still punish the defendant
and not let the judgment against him be less just
because the plaintiff has, say, insurance.To make
sure the plaintiff doesn’t double dip, the court
allows through subrogation insurance companies

66
67

and other parties to collect finances they have paid


out from the money awarded in a civil case.

3. Zimmerman v. Ausland
a. Court looks at whether a plaintiff who failed to get a
knee surgery after an accident involving the
negligent defendant was liable for the further
damage to her knee.
b. The court uses the “reasonable prudent person”
test to determine if the plaintiff was right or
wrong in attempting to mitigate the damages of
an injury.

4.

67

Potrebbero piacerti anche